Company: L&T

Total 90 questions , 90 minutes  —SECTIONAL CUTOFF IS THERE
1.  Aptitude     (30 quest)
2.  Reasoning  (30 quest)
3.  English       (30 quest)
1.  Aptitude     (30 quest)
1. The average salary of 3 workers is 95 Rs. per week. If one earns Rs.115 and second earns Rs.65 how much is the 
    salary of the 3rd worker.
    Ans.105.
2. A 16 stored building has 12000 sq.feet on each floor. Company A rents 7 floors and company B rents 4 floors. 
    What is the number of sq.feet of unrented floor space. 
    Ans.60000

3.During a given week A programmer spends 1/4 of his time preparing flow chart, 3/8 of his time coding and the 
    rest of the time in debugging the programs. If he works 48 hours during the week , how many hours did he spend 
    debugging the program.
    Ans. 18.
4. A company installed 36 machines at the beginning of the year. In March they installed 9 additional machines and 
    then disconnected 18 in August. How many were still installed at the end of the year. 
    Ans. 27

5. A man owns 2/3 of the market research beauro business and sells 3/4 of his shares for Rs. 75000. What is the  
    value of Business ? 
    Ans.150000
6. If 12 file cabinets require 18 feet of wall space, how many feet of wall space will 30 cabinets require?
    Ans.45

7. A computer printer produced 176,400 lines in a given day. If the printer was in operation for seven hours during the 
    day, how many lines did it print per minute?
    Ans.420
8. From its total income, A sales company spent Rs.20,000 for  advertising, half of the remainder on commissions and 
    had Rs.6000 left. What was its total income?
    Ans.32000
9. On Monday a banker processed a batch of cheques, on Tuesday she processed three times as many, and on 
     Wednesday she processed 4000 cheques. In the three days, she processed 16000 cheques. How many did she 
     process on Tuesday?
    Ans.9000

10. The cost of four dozen proof machine ribbons and five dozen accouting machine ribbons was Rs.160/-. If one 
      dozen accounting machine ribbons cost Rs.20/-, what is the cost of a dozen proof machine ribbons?
     Ans.Rs.15

11. If a clerk can process 80 cheques in half an hour, how many cheques can she process in a seven and one half    
       hour day?
     Ans.1200

12. In a library, there are two racks with 40 books per rack. On a given day, 30 books were issued. What fraction 
      remained in the racks?
      Ans.5/8

13. The average length of three tapes is 6800 feet. None of the tapes is less than 6400 feet. What is the greatest 
      possible length of one of the other tapes?
     Ans.7600

14. A company rented a machine for Rs.700/- a month. Five years later the treasurer calculated that if the company 
      had purchased the machine and paid Rs.100/- monthly maintenance charge, the company would have saved 
      Rs.2000/-. What was the purchase price of the machine?
      Ans.Rs.34000

15. Two computers each produced 48000 public utility bills in a day. One computer printed bills at the rate of 9600 an 
      hour and the other at the rate of 7800 an hour. When the first computer finished its run, how many bills did the 
      other computer still have to print?
       Ans.9000

16. If a salesman’s average is a new order every other week, he will break the office record of the year. However, 
      after 28 weeks, he is six orders behind schedule. In what proportion of the remaining weeks does he have to 
       obtain a new order to break the record?
      Ans.3/4
17 On a given day, a bank had 16000 cheques returned by customers. Inspection of the first 800 cheques indicated 
      that 100 of those 800 had errors and were therefore the available immediately for data processing. On this basis,   
      how many cheques would be available immediately for data processing on that day?
      Ans.14000

18. A tape manufacturer reduces the price of his heavy duty tape from Rs.30/- to Rs.28/- a reel and the price of a 
      regular tape from Rs.24/- to Rs.23/- a reel. A computing centre normally spends Rs.1440/- a month for tapes    
      and 3/4 of this is for heavy duty tapes. How much will they save a month under the new prices?

          Ans.Rs.87

19. The dimensions of a certain machine are 48″ X 30″ X 52″. If the size of the machine is increased proportionately 
      until the sum of its dimensions equals 156″, what will be the increase in the shortest side?
       Ans. 6″
20 In a certain company, 20% of the men and 40% of the women attended the annual company picnic. If 35% of all 
      the employees are man, what percent of all the employees went to the picnic?
      Ans.33%

21. It cost a college Rs.0.70 a copy to produce a Programme for the homecoming football game. If Rs.15,000/- was 
      received for advertisements in the programme, how many copies at Rs.0.50 a copy must be sold to make a profit 
      of Rs.8000/- ?
      Ans. 35000
22. If the digits of my present age are reversed then i get the age of my son. If 1 year ago my age was twice as that 
      of  my son. Find my present age.
       Ans. father-73, son-37
23. There are 6561 balls out of them 1 is heavy. Find the min. no. of times the balls have to be weighed for finding out 
      the heavy ball.
      Ans. 8

24.If I walk with 30 miles/hr i reach 1 hour before and if i walk with 20 miles/hr i reach 1 hour late. Find the distance 
     between 2 points and the exact time of reaching destination is 11 am then find the speed with which it walks.
     Ans. 120miles and 24 miles/hr

25. A thief steals half the total no of loaves of bread plus 1/2 loaf from a bakery. A second thief steals half the 
      remaining no of loaves plus 1/2 loaf and so on. After the 5th thief has stolen there are no more loaves left in the 
      bakery. What was the total no of loaves did the bakery have at the beginning.
    Ans: 31.
26. A person needs 6 steps to cover a distance of one slab. If he increases his foot length (step length) by 3 inches he 
      needs only 5 steps to cover the slabs length. What is the length of the each slab.
    Ans: 31 inches.
27. There are 19 red balls and one black ball. Ten balls are put in one jar and the remaining 10 are put in another jar. 
       What is the possibility that the black is in the right jar.
        Ans: 1/2.
28. There is one lily in the pond on 1st june . There are two in the pond on 2nd june . There are four on 3rd june and 
      so on. The pond is full with lilies by the end of the june .
    (i) On which date the pond is half full?
    Ans: 29th.
29. A lorry starts from Banglore to Mysore at 6.00 A.M, 7.00 A.M, 8.00 am…..10 pm. Similarly one another starts 
       from Mysore to Banglore at 6.00 am,7.00 am, 8.00 am…..10.00pm. A lorry takes 9 hours to travel from 
       Banglore to Mysore and vice versa.

      (i) A lorry which has started at 6.00 am will cross how many lorries.
       Ans: 10.
30. A person meets a train at a railway station coming daily at a particular time . One day he is late by 25 minutes, and 
      he meets the train 5 k.m. before the station. If his speed is 12 kmph, what is the speed of the train ?
       Ans: 60 kmph
2.  Reasoning (30 quest)
Answer Questions 1 to 5 on the basis of the information given below:
The only people to attend a conference were four ship captains and the first mates of three of those captains.  The captains were L, M, N and O; the first mates were A, D and G Each person in turn delivered a report to the assembly as follows:
Each of the first mates delivered their report exactly after his or her captain. The first captain  to speak was M, and captain N spoke after him.
1. Among the following which is not an appropriate order of delivered reports?
A.M, A, N, G, O, L, D
B.M, D, N, G, L, O, A
C.M, N, A, L, D, O, G
D.M, N, A, O, D, L, G
E.M, N, G, D, O, L, A
Ans : E
2. In case L speaks after A, and A is the third of the first mates to speak, then among the following statements which would be untrue?
A.O spoke immediately after G.
B.The order of the first four speakers was M, G, N, D.
C.O’s first mate was present.
D.A was the fourth speaker after M.
E.The captains spoke in the order M, N, O, L.
Ans : D
3. Among the following statements which statement must be true?
A.In case the second speaker was a captain, the seventh speaker was a first mate.
B.In case the second speaker was a first mate, the seventh speaker was a captain.
C.In case the third speaker was a first mate, the seventh speaker was a captain.
D.In case the third speaker was a captain, the seventh speaker was a first mate.
E.In case the seventh speaker was a first mate, the first and third speakers were captains.
Ans : A
4. In case A spoke immediately after L and immediately before O, and O was not the last speaker, L spoke
A.second
B.third
C.fourth
D.fifth
E.sixth
Ans : C
5. In case G is M’s first mate, D could be the person who spoke immediately
A.prior to T
B.prior to L
C.prior to V
D.after T
E.after V
Ans : D
6. At the college entrance exam, a candidate is admitted according to whether he has passed or failed the test. Of the 
    candidates who are really capable, 80 % pass the test and of the incapable, 25 % pass the test. Given that 40 % of 
    the candidates are really capable, then the proportion of the really capable students who can pass the test to the 
    total students who can pass is about:
   A. 68%     B. 70%   C. 75%     D. 73%
    Ans: B
7. Excluding stoppages, the speed of a bus is 54 km/hr and including stoppages, it is 45 km/hr. For how many minutes 
    does the bus stop per hour ?
    A. 12      B. 10      C. 9       D. 15
    Ans: B
8. A boy goes to see a film and finds a man who is his relative. The man is the husband of the sister of his mother. How 
    is the man related to the boy?
    A. Uncle    B. Brother    C. Nephew    D. None of the above
    Ans: A
9. The ratio between the number of passengers travelling by I & II Class between two railway stations is 1 : 50, 
    whereas the ratio of the I & II Class fares between the same stations is 3 : 1. If on a particular day, Rs. 1325 were 
    collected from the passengers travelling between these stations, what was the amount collected from the II Class 
    passengers ?
    A. Rs.1250        B. Rs. 1100        C. Rs. 1000        D. Rs.1150
 
    Ans: A
10. When Rajeev was born, his father was 32 years older than his brother and his mother was 25 years older than    
      his sister. If Rajeev’s brother is 6 years older than Rajeev and his mother is 3 years younger than his father, how 
      old was Rajeev’s sister when Rajeev was born?
     A. 15 Years       B. 7 Years        C. 17 Years        D. 10 Years
     Ans: D
11. If STRAY is coded as TUSBZ then how will MOURN be coded?
(A) LPVSO
(B) NPVSO
(C) NVPSO
(D) NPSVO
(E) None of these
Ans : (B)
12. If MAGIC is coded as PXJFF then how will LEASH be coded?
(A) PBDVD
(B) OBDVL
(C) OHEOK
(D) OBDPK
(E) None of these
Ans : (D)
13. If DOWN is coded as FQYP then how will WITH be coded?
(A) KYN
(B) JYK
(C) YKVJ
(D) JKVY
(E) None of these
Ans : (C)
14. If BINARY is coded as DHPZTK then how will KIDNAP be coded?
(A) MKFPCQ
(B) MHFPZQ
(C) IKFMYO
(D) MHFMCO
(E) None of these
Ans : (D)
15. If RASCAL is coded as QZRBZK then how will SOLDER be coded?
(A) RNMEDQ
(B) RPKEDS
(C) TPMEFS
(D) RNKCDQ
(E) None of these
Ans : (D)
Directions 16:20 : In each question below are given two statements followed by two conclusions numbered  I and II.  You have to take the given two statements to be true even if they seem to be at variance from commonly known facts. Read the conclusions and then decide which of the given conclusions logically follows from the two given statements, disregarding commonly known facts.
Give answer (A) if only conclusion I follows; (B) if only conclusion II follows; (C) if either I or II follows (D) if neither I or II follows and (E) if both I and II. 
16.Statements : All puppets are dolls
                      All dolls are toys
  Conclusions : I. Some toys are puppets
                       II. All toys are puppets
    Ans: A
17.Statements : All apples are oranges
                      Some oranges are papayas
  Conclusions : I. Some apples are papayas
                       II. Some papayas are apples
    Ans: D
18. Statements :Some players are singers
                      All singers are tall
  Conclusions : I. Some players are tall
                       II. All players are tall
    Ans: A
19.Statements : All coins are crows
                       Some crows are pens
  Conclusions : I. No pen is coin
                       II. Some coins are pens
    Ans: D
20.Statements : All men are married
                        Some men are educated
  Conclusions : I. Some married are educated
                       II. Some educated are married
    Ans: E
21. Pointing to a man Snehlata says, “He is the only son of my father’s father.” How is Snehlata related to the man ?
(A) Mother
(B) Grand daughter
(C) Niece
(D) Sister
(E) None of these
Ans : (E)
22. Pointing to a lady in photograph, Madhurendra said, “Her mother is the only daughter of my mother’s mother.” 
      How is Madhurendra related to the lady?
(A) Nephew
(B) Uncle
(C) Maternal uncle
(D) Brother
(E) None of these
Ans : (D)
23. If P x Q means— ‘P is the brother of Q’, P ÷ Q means ‘P is the son of Q’ and ‘P – Q’ means ‘P is the sister of Q’ , 
      then which of the following relations will show that Q is the maternal uncle of P ?
(A) Q x R ÷ P
(B) Q ÷ R – P
(C) P x R – Q
(D) P ÷ R – Q
(E) None of these
Ans : (D)
24. In the class of 40 students, if Sanju is at 30th place from one end, what is his position from the other end?
(A) 9th
(B) 12th
(C) 10th
(D) 11th
(E) None of these
Ans : (D)
25. In a row of girls, Nivedita is 15th from the left and Vimla is 23rd from the right. If they interchange their positions, 
     then Nivedita becomes 18 th from the left. Then at what position will Vimla be from the right?
(A) 25th
(B) 24th
(C) 26th
(D) 20th
(E) None of these
Ans : (C)
26. As ‘Hindu worshiper’ is related to ‘Temple’ in the same way ‘Maulvi’ is related to what?
(A) Monastery
(B) Church
(C) Mosque
(D) Sikh temple
(E) None of these
Ans : (C)
27. As ‘Hungry’ is related to ‘Food’ in the same way ‘Thirsty’ is related to what?
(A) Drink
(B) Tea
(C) Coffee
(D) Juice
(E) Waler
Ans : (E)
28. As ‘Fly’ is related to ‘Parrot’ in the same way ‘Creep’ is related to what?
(A) Snake
(B) Rabbit
(C) Fish
(D) Crocodile
(E) Sparrow
Ans : (A)
29.  If ‘M’ denotes ‘÷’, ‘K, denotes ‘–’, ‘T’ denotes ‘×’ and ‘R’ denotes ‘+’, then;    20 K 16 T 8 M 4 R 6 = ?
(A) 18
(B) – 6
(C) – 12
(D) – 18
(E) None of these
Ans: B
30. How many such pairs of letters are there in the word TERMINATE each of which has as many letters between them in the word as in the English alphabet ?
(A) None
(B) One
(C) Two
(D) Three
(E) More than three
            Ans: B
3.  English      (30 quest)
Directions 1-5: In each of the following questions, there is a certain relationship between two given words on one side of : : and one word is given on another side of : :while another word is to be found from the given alternatives, having the same relation with this word as the words of the given pair bear. Choose the correct alternative.
1.Oxygen : Burn : : Carbon dioxide : ?
    (A) Isolate     (B) Foam     (C) Extinguish     (D) Explode
    Ans: (C)
2. Grain : Stock : : Stick : ?
    (A) Heap     (B) Bundle     (C) Collection     (D) String
    Ans: (B)
3. Planet : Orbit : : Projectile : ?
    (A) Trajectory     (B) Track     (C) Milky way     (D) Path
    Ans: (A)
4. Genuine : Authentic : : Mirage : ?
(A) Image     (B) Transpiration     (C) Reflection     (D) Illusion
Ans: (D)
5.Illiteracy : Education : : Flood : ?
(A) Rain     (B) Bridge      (C) Dam     (D) River
Ans: (C)
Directions for Questions 6 to10 :Each of the following sentences has been divided into four parts.  There is an error in one of the parts. Point out the part which has an error .
 6.A) I was astonished by the highly exciting tricks / 
    B) the acrobat displayed on the rope /
    C) at great risk of like
    D) at such an advanced age.
      Ans: A
7.  A) We, who are fortunate enough / 
    B) to have lived in the present century, /
    C) hardly realize how our ancestors suffered /
    D) from the belief in the existence of evil spirits.
       Ans: B
8. A) We were expecting at least twenty delegates /
    B) to participate in the seminar, /
    C) but when I reached the hall,
    D) I found no any delegate present there.
      Ans: D
9. A) We bought five dozens pencils from a shop,/
    B) but on opening in the packet at home /
    C) we found five pencils short, /
    D) but the shopkeeper did not accept the responsibility of the shortage.
     Ans: A
10. A) I wanted to book   a  parcel weighting twenty and a half kilos /
     B) but the booking -clerk refused to book it /
     C) on the ground
     D) that it was not properly sealed.
    Ans: A
Directions for Questions 11-15: In each of the following questions, a paragraph or a sentence has been broken up into different parts. The parts have been scrambled and numbered as given below.  Choose the correct order of these parts from the given alternatives
11.A. The potential exchanges between the officials of IBBF and the Maharashtra Body-Building Association has all the trappings of a drama we are accustomed to.
B. In the case of sportspersons, there is room for some sympathy, but the apathy of the administrators, which has even led to sanctions from international bodies, is unpardonable.
C. A case in the point is the hefty penalty of US $10,000 slapped on the Indian Body-Building Federation for not fulfilling its commitment for holding the Asian Championships in Mumbai in October.
D. It is a matter of deep regret and concern that the sports administrators often cause more harm to the image of the country than sportsmen and sportswomen do through their dismal performances.

1] CABD   2] DBCA   3] DABC   4] CDBA

Ans: 2
12.A. Its cargo consisted of 38 sacks of spices and Magellan himself had been hacked to pieces on the beach of Mactan in the Phillipines
B. So contrary to popular beliefe it was the crew of the Victoria who were the first men to have sailed around the globe
C. In spetembre 1522 Victoria , the sole survivor of the Armada, limped into the spanish port San Lucar , manned by a skeleton crew of 15, so weak they could not talk
D. In septembre 1519 the Armada de Molucca of five ships and 250 sailors has set out from San lucar de Barrameda under the command of Fernando de Magellan
E. It was to sail to the spice islands of the Malayan Archipelago where they were to excahnge an assortemnt of bells , mirrors , and scissors for cinnamon and cloves.

1] DECAB   2] AEDCB   3] CDEAB   4] DEABC

 Ans:1
13.A. What came out was very large garland made out of currency notes.
B. The unsuspecting governor opened the box in full view of the gathering
C. When the RBI governor came to inaugrate the new printing press , the local unit of the BJP handed him a gift wrapped box
D. There was a twist – the notes were all as tattered as notes could get

1] DACB  2] CABD   3] CBAD   4] DCAB

Ans:3
14.A. But in the industrial era destroying the enemy’s productive capacity means bombing the factories which are located in the cities.
B. So in the agrarian era, if you need to destroy the enemy’s productive capacity, what you want to do is bum his fields, or if you’re really vicious, salt them.
C. Now in the information era, destroying the enemy’s productive capacity means destroying the information infrastructure.
D. How do you do battle with your enemy?
E. The idea is to destroy the enemy’s productive capacity, and depending upon the economic foundation, that productive capacity is different in each case.
F. With regard to defence, the purpose of the military is to defend the nation and be prepared to do battle with its enemy.

1] FDEBAC   2] FCABED   3] DEBACF   4] DFEBAC

Ans:1
15.A. The situations in which violence occurs and the nature of that violence tends to be clearly defined at least in theory, as in the proverbial Irishman’s question: ‘Is this a private fight or can anyone join in?’
B. So the actual risk to outsiders, though no doubt higher than our societies, is calculable.
C. Probably the only uncontrolled applications of force are those of social superiors to social inferiors and even here there are probably some rules.
D. However binding the obligation to kill, members of feuding families engaged in mutual massacre will be genuinely appalled if by some mischance a bystander or outsider is killed.

1] DABC   2] ACDB   3] CBAD   4] DBAC

Ans:1
Directions for Questions 16 to 20: The passage given below is followed by a set of five questions. Choose the most appropriate answer to each question.
To summarize the Classic Maya collapse, we can tentatively identify five strands. I acknowledge, however, that Maya archaeologists still disagree vigorously among themselves—in part, because the different strands evidently varied in importance among different parts of the Maya realm; because detailed archaeological studies are available for only some Maya sites; and because it remains puzzling why most of the Maya heartland remained nearly empty of population and failed to recover after the collapse and after re-growth of forests.

With those caveats, it appears to me that one strand consisted of population growth outstripping available resources: a dilemma similar to the one foreseen by Thomas Malthus in 1798 and being played out today in Rwanda (Chapter 10), Haiti (Chapter 11), and elsewhere. As the archaeologist David Webster succinctly puts it, “Too many farmers grew too many crops on too much of the landscape.” Compounding that mismatch between population and resources was the second strand: the effects of deforestation and hillside erosion, which caused a decrease in the amount of useable farmland at a time when more rather than less farmland was needed, and possibly exacerbated by an anthropogenic drought resulting from deforestation, by soil nutrient depletion and other soil problems, and by the struggle to prevent bracken ferns from overrunning the fields.

The third strand consisted of increased fighting, as more and more people fought over fewer resources. Maya warfare, already endemic, peaked just before the collapse. That is not surprising when one reflects that at least 5,000,000 people, perhaps many more, were crammed into an area smaller than the state of Colorado (104,000 square miles). That warfare would have decreased further the amount of land available for agriculture, by creating no-man’s lands between principalities where it was now unsafe to farm. Bringing matters to a head was the strand of climate change. The drought at the time of the Classic collapse was not the first drought that the Maya had lived through, but it was the most severe. At the time of previous droughts, there were still uninhabited parts of the Maya landscape, and people at a site affected by drought could save themselves by moving to another site. However, by the time of the Classic collapse the landscape was now full, there was no useful unoccupied land in the vicinity on which to begin anew, and the whole population could not be accommodated in the few areas that continued to have reliable water supplies.

As our fifth strand, we have to wonder why the kings and nobles failed to recognize and solve these seemingly obvious problems undermining their society. Their attention was evidently focused on their short-term concerns of enriching themselves, waging wars, erecting monuments, competing with each other, and extracting enough food from the peasants to support all those activities. Like most leaders throughout human history, the Maya kings and nobles did not heed long-term problems, insofar as they perceived them. We shall return to this theme in Chapter 14.

Finally, while we still have some other past societies to consider in this book before we switch our attention to the modern world, we must already be struck by some parallels between the Maya and the past societies discussed in Chapters 2-4. As on Easter Island, Mangareva, and among the Anasazi, Maya environmental and population problems led to increasing warfare and civil strife. As on Easter Island and at Chaco Canyon, Maya peak population numbers were followed swiftly by political and social collapse. Paralleling the eventual extension of agriculture from Easter Island’s coastal lowlands to its uplands, and from the Mimbres floodplain to the hills, Copan’s inhabitants also expanded from the floodplain to the more fragile hill slopes, leaving them with a larger population to feed when the agricultural boom in the hills went bust. Like Easter Island chiefs erecting ever larger statues, eventually crowned by pukao, and like Anasazi elite treating themselves to necklaces of 2,000 turquoise beads, Maya kings sought to outdo each other with more and more impressive temples, covered with thicker and thicker plaster— reminiscent in turn of the extravagant conspicuous consumption by modern American CEOs. The passivity of Easter chiefs and Maya kings in the face of the real big threats to their societies completes our list of disquieting parallels.
16. According to the passage, which of the following best represents the factor that has been cited by the author in the context of Rwanda and Haiti?
(1) Various ethnic groups competing for land and other resources.
(2) Various ethnic groups competing for limited land resources.
(3) Various ethnic groups fighting wit each other.
(4) Various ethnic groups competing for political power.
(5) Various ethnic groups fighting for their identity.
Ans: 2
17. By an anthropogenic drought, the author means
(1) A drought caused by lack of rains.
(2) A drought caused due to deforestation.
(3) A drought caused by failure to prevent bracken ferns from overrunning the fields.
(4) A drought caused by actions of human beings.
(5) A drought caused by climate changes.

Ans: 4
18. According to the passage, the drought at the time of Maya collapse had a different impact compared to the droughts earlier because
(1) The Maya kings continue to be extravagant when common people were suffering.
(2) It happened at the time of collapse of leadership among Mayas.
(3) It happened when the Maya population had occupied all available land suited for agriculture.
(4) AIt was followed by internecine warfare among Mayans.
(5) Irreversible environmental degradation led to this drought.
Ans: 3
19. According to the author, why is it difficult to explain the reasons for Maya collapse?
(1) Copan inhabitants destroyed all records of that period.
(2) The constant deforestation and hillside erosion have wiped out all traces of the Maya kingdom.
(3) Archaeological sites of Mayas do not provide any consistent evidence.
(4) It has not been possible to ascertain which of the factors best explains as to why the Maya civilization collapsed.
(5) At least five million people were crammed into a small area.
Ans: 4
20. Which factor has not been cited as one of the factors causing the collapse of Maya society?
(1) Environmental degradation due to excess population.
(2) Social collapse due to excess population.
(3) Increased warfare among Maya people.
(4) Climate change.
(5) Obsession of Maya population with their own short-term concerns.
          Ans: 3
Directions 21-25: Pick out from the words, given below each sentence, the  word which would  complete the sentence correctly and meaningfully.
21. Unhygienic circumstances………………health problems.
    A) give rise to    B) bring in to existence    C) call for    D) replace
     Ans: A
22. If she were chosen for the job, she……….a good secretary.
    A) will make    B) can make    C) would make    D) would have made
     Ans: C
23. Didn’t you go to the restaurant yesterday………..?
    A) No I did’nt    B) Yes I didn’t    C) No, I did    D) will, yes.
      Ans: A
24. The Chernobil tragedy had taken place about four years………….
    A) past    B) ago    C) before    D) since
    Ans: B
25.Every government servant is finding it necessary to supplement his…………… by doing something extra.
    A) earnings    B) profits    C) remuneration    D) income
    Ans: D
Directions 11-15: In each of the following questions, out of the given alternatives, choose the one which       best expresses the meaning of the given word.
26. Abolish
A.  end   
B.  minimize
C. prevent
D.  denounce
            Ans: A
27. make up
A.invent
B.prepare
C.replace
D.complete
            Ans: D
28. dispatched
A.taken by force
B.ordered to go
C.expelled
D.sent
           Ans: D
29. ARDUOUS
A.Short
B.Difficult
C.Easy
D.Expensive
        Ans: B
30. ABODE
A.Family
B.Farm
C.Car
D.Home

Total 90 questions , 90 minutes  —SECTIONAL CUTOFF IS THERE
1.  Aptitude     (30 quest)
2.  Reasoning (30 quest)
3.  English      (30 quest)
1.  Aptitude     (30 quest)
1. If the thrice of three consecutive odd no is equal to the more  three of twice the last no. then find the 3rd               
    (largest odd no).?
     Ans: 11.
2. Rahul 12th from right and 4th from left . How many men should be added in the queue to make a group of 28?
     Ans 13 

3. Present population of town is 35,000 having males and females. If The population of males is increased by 6%    
    and if the population of females is increased by 4%, then after 1 year the population becomes 36,700.
    Find the number males and females ? 
   Ans18,000 
4.Gets 90% of B. B gets 25% more than C. C gets 80% of D. Marks of A are given then find the percentage marks    
    obtained by D. 
    Ans = 80% 

5. Length of rectangular plot is 3 times its breadth having area 3 hectare . A man walks on its perimeter at 4 km/hr .
    Find the time required ?
    Ans 12 min
6  Speed of trains A and B are 29 & 56 km/hr travelling in the same direction. Man in slower train passes in 16 min. 
    Find the length of the train.? 
    Ans 120m 
7 Y catches 5 times more fishes than X. If total number of fishes caught by X and Y is 48, then number of fishes  
    caught by X? 
    Ans: 8 

8 The girl’s age is twice that of boy, if the boy is four years old. After four years the age of the girl is ?
    Ans: 12 years 
9  A clock is late by 1 minute 27 seconds in a month. Then how much will it be late in 1 day? 
    Ans: 2.9 seconds 
10 At 20% discount, a cycle is sold at a selling price of 2500 Rs. What is the actual price? 
     Ans: Rs. 3125 
11. A man purchased 6 stamps of rupees 1 and seven stamps of 50 paise. he paid Rs. 12. how much change              
      he got back ?
      Ans. 2.50 Rs
12. 8 man work for 6 days to complete a work. How many men are required to complete same work in 1/2 day. 
      Ans. 96 men.
13. A man drives at a speed of 40 miles/hr. His wife left 30 mins. late with 50 miles/hr speed. when will they meet ? 
      Ans. 2 hours
14. 1,40,00,000 pencils are put up straight. all the pencils are of length range 3 to 6 inches. 80% of the pencils have 
       average of five inches. so the find out the total length spanned by the pencils.
       Ans. 1000 to 1500 miles.

15. One person says ‘his only daughter is my son’s mother ’, what is the relationship between 1st &2nd person. 
     Ans :son –in –law

16  Alphabet ‘A’ to ‘Z’. If we arrange in reverse order (Z to A) then which letter will be in the 5th position towards 
      right ,taken from 11th better from left. 
      Ans: k

17.  In a railway station, there are two trains going. One in the harbor line and one in the main line, each having a 
       frequency of 10 minutes. The main line service starts at 5 o’clock and the harbor line starts at 5.02A.M. A  
       man goes to the station every day to catch the first train that comes. What is the probability of the man catching 
       the first train? 
       Ans: 0.8

18. Light glows for every 13 seconds . How many times did it glow between 1:57:58 and 3:20:47 am.
      Ans : 383 + 1 = 384 

19 From a vessel, 1/3rd of the liquid evaporates on the first day. On the second day 3/4th of the remaining liquid 
     evaporates. What fraction of the volume is present at the end of the second day.
     Ans: 50% 

20. There are 20 poles with a constant distance between each pole. A car takes 24 second to reach the 12th pole.
      How much will it take to reach the last pole. 
      Ans: 41.45 seconds
21. Two trains starting at same time, one from Bangalore to Mysore and other in opposite direction arrive at their 
     destination 1 hr and 4 hours respectively after passing each other. How much faster is one train from other?
     Ans: Twice
22. Two trains start from stations A and B spaced 50kms apart at the same time and speed. As the trains start,            
      a bird flies from one train towards the other and on reaching the second train, it flies back to the first train. This is 
      repeated till the trains collide. If the speed of the trains is 25 km/h and that of the bird is 100km/h. How much did 
      the bird travel till the collision.
      Ans: 100kms
23. Sometime after 10:00 PM a murder took place. A witness claimed that the clock must have stopped at the time of 
      the shooting. It was later found that the position of both the hands were the same but their positions had   
      interchanged Tell the time of the shooting (both actual and claimed).
     Ans: Time of shooting = 11:54 PM Claimed Time = 10:59 PM 

24. 500 men are arranged in an array of 10 rows and 50 columns according to their heights. Tallest among each row 
      of all are asked to fall out. And the shortest among them is A. Similarly after resuming that to their original  
      positions  that the shortest among each column are asked to fall out. And the tallest among them is B. Now who is 
      taller among A and B ? 
      Ans. A
25. A family X went for a vacation. Unfortunately it rained for 13 days when they were there. But whenever it rained  
      in the mornings, they had clear afternoons and vice versa. In all they enjoyed 11 mornings and 12 afternoons. 
      How many days did they stay there totally? 
      Ans: 18
26. The minute and the hour hand of a watch meet every 65 minutes. How much does the watch lose or gain time and 
      by how much? 
      Ans: Gains; 5/11 minutes 

27. Every day a cyclist meets a train at a particular crossing. The road is straight before the crossing and both are 
      traveling in the same direction. The cyclist travels with a speed of 10kmph. One day the cyclist comes late by 25 
      min. and meets the train 5km before the crossing. What is the speed of the train?
      Ans: 60kmph 
28. A ship went on a voyage. After it had traveled 180 miles a plane started with 10 times the speed of the ship.
      Find the distance when they meet from starting point. 
      Ans: 200miles.
29. Father’s age is three years more than three times the son’s age. After three years, father’s age will be ten years 
      more than twice the son’s age. What is the father’s present age? 
      Ans: 33 years. 
30. A man collects cigarette stubs and makes one full cigarette with every 8 stubs. If he gets 64 stubs how many full 
      cigarettes can he smoke ?  
       Ans: 8+1=9 

2.  Reasoning (30 quest)
Answer Questions 1 to 5 on the basis of the information given below:

K, L, M, N, P, Q, R, S, U and W are the only ten members in a department. There is a proposal to form a team from within the members of the department, subject to the following conditions:

A team must include exactly one among P, R, and S.
A team must include either M or Q, but not both.
If a team includes K, then it must also include L, and vice versa.
If a team includes one among S, U, and W, then it must also include the other two.
L and N cannot be members of the same team.
L and U cannot be members of the same team.

The size of a team is defined as the number of members in the team.

What could be the size of a team that includes K?
A. Only 2        B. 3 or 4         C. 2 or 3        D. Only 4        E. 2 or 4
Ans: D
2. In how many ways a team can be constituted so that the team includes N?
A. 5        B. 6         C. 4        D. 2        E. 3
Ans: B
 3. What would be the size of the largest possible team?
A. cannot be determined        B. 6         C. 7        D. 8        E. 5
Ans: E
4.  Who can be a member of a team of size 5?
A. R        B. L         C. P        D. M        E. K
Ans: D
 5. Who cannot be a member of a team of size 3?
A. P        B. L         C. M        D. N        E. Q
Ans: B
6. In an island three persons are there Jam ,Dam and Sam . sam having pencil, dam having cap and Jam having book .
    we have find who is author among them if.
    (1)Sam says ,Jam is author but author wearing cap.
    (2)Dam says ,I am author ,but author not having cap.
    (3)Sam says, I am author ,but author having note book.
    Find who is author?

    Ans : jam 
7. My Dad has a miniature Pyramid of Egypt. It is 3 inches in height. Dad was invited to display it at an exhibition.  
    Dad felt it was too small and decided to build a scaled-up model of the Pyramid out of material whose density is    
    (1 / 7) times the density of the material used for the miniature. He did a “back-of-the-envelope” calculation to check 
    whether the model would be big enough. If the mass (or weight) of the miniature and the scaled-up model are to be 
    the same, how many inches in height will be the scaled-up Pyramid?
    A. 5.78        B.5.82        C. 5.68        D. 5.74
    Ans D
8. Neelam rides her bicycle from her house at A to her club at C, via B taking the shortest path. Then the number of 
    possible shortest paths that she can choose is
A.1170
B.630
C.792
D.1200
E.936
     Ans: A
9. In a triangle ABC, the lengths of the sides AB and AC equal 17.5 cm and 9 cm respectively. Let D be a point on 
    the line segment BC such that AD is perpendicular to BC. If AD=3 cm, then what is the radius (in cm) of the circle 
    circumscribing the triangle ABC?
A.17.05
B.27.85
C.22.45
D.32.25
E.26.25
    Ans: E
10. Gary wants to dry his clothes. It takes him 60 minutes to dry 10 clothes under the sun. The very next day, he has 
      20 clothes (double the numbers) to dry the clothes. What should be the expected time to dry double the number 
      of clothes?
      Ans: 60 minutes
Answer Questions 11 to1 5 on the basis of the information given below:
There are 6 women Shalini, Divya, Ritu, Rashmi, Nisha and Renu in a family of twelve members. There are few married couples in the family and none of the grandchildren are married. Sunil is married in to the family. Rohan, Mahesh and Jatin have a nephew Dipesh who is the only son of Rashmi. Ravi is the paternal grandfather of Nisha. Ritu is the daughter-in-law of Shalini. Renu is the first cousin of Dipesh. Shalini has only three Grandchildren. Mahesh has two borthers and only one sister Rashmi and a sister-in-law Divya. Dipesh’s only unmarried maternal uncle, Jatin is the brother-in-law of Sunil. Rohan is the paternal uncle of Nisha. Ritu has two daughters one of whom is Nisha.

11. Nisha is:
A. none of these            B. Rohan’s daughter        C. Renu’s cousin         D. Jatin’s mother
Ans: A

12. Dipesh is:
A. Ravi’s grandson            B. Sunil’s nephew       C. Mahesh’s son         D. Rohan’s son
Ans: A

13. How many married couples are there in the second generation?
A. 1        B. 3       C. 4         D. 2
Ans: B
14. Which one of the following is true?
A. Ravi has only Two Married children.        B. None of these.      
 C. Dipesh is Mahesh’s son                           D. Ravi is the paternal grandfather of Renu
Ans: D

15. Which one of these is a married couple?
A. Rohand and Ritu        B. Mahesh and Ritu      C. Renu and Sunit         D. Shalini and Mahesh
Ans: B 
16. If KEDGY is coded as EKDYG then how will LIGHT be coded ?
        (A) ILHTG
        (B) ILGHT
        (C) ILGTH
        (D) THGIL
        (E) None of these
        Ans : (C)

17. If RAVE is coded as SXWB then how will SCAW be coded ?
        (A) TDBO
        (B) TZBK
        (C) PZXK
        (D) TVXK
        (E) None of these
        Ans : (B)

18. If SPANK is coded as PSNAK then how will THROW be coded?
        (A) HTORW
        (B) HTWOR
        (C) HTWRO
        (D) HTRWO
        (E) None of these
        Ans : (A)

19. If UDOMETER is coded as DUMOTERE then how will SUBLEASE be coded?
        (A) USLBESAE
        (B) USLBAEES
        (C) USBAELES
        (D) USLBEAES
        (E) None of these
        Ans : (B)

20. If PURSER is coded as UPSRRE then how will PERIODIC be coded?
        (A) EPRIDOIC
        (B) PEIRDOCI
        (C) EPIRDOCI
        (D) EPIRODCI
        (E) None of these
        Ans : (C)

21 In a row of ladies Manorama is 20th from the right and Kanta is 10th from the left. When they interchange their 
     positions Manorama becomes 25th from the right. What is the total number of ladies in the row ?
        (A) 35
        (B) 34
        (C) 44
        (D) 24
        (E) None of these
        Ans : (B)

22. A number of people are standing in a row in which Kailash is 20th from the left and Hemant is 25th from the right. 
     If they interchange their positions then Kailash becomes 25th from the right. How many people are there in the 
     row?
(A) 49
(B) 44
(C) 45
(D) Data inadequate
(E) None of these
Ans : (D)
23. In a row of boys, Udai is 23rd from the left and Ashok is 8th from the right. When they interchange their positions Udai becomes 18th from the right. What will be the position of Ashok from the left ?
    (A) 15th
    (B) 10th
    (C) 40th
    (D) Data inadequate
    (E) None of these
    Ans : (D)
24 A lady pointing to a woman in a photograph says, “She is the only daughter of my father-in-law.” How is the woman related to the lady?
    (A) Mother
    (B) Sister
    (C) Friend
    (D) Aunt
    (E) None of these
    Ans : (E)

25. Pointing to a woman in a photograph a man says, “Her mother’s mother is the mother of my father.” How is the man related to the woman in a photograph?
    (A) Uncle
    (B) Maternal cousin
    (C) Nephew
    (D) Grand son
    (E) None of these
    Ans : (B)
Directions 26-30 : In each question below are given two statements followed by two conclusions numbered I and II.  You have to take the given two statements to be true even if they seem to be at variance from commonly known facts. Read the conclusions and then decide which of the given conclusions logically follows from the two given statements, disregarding commonly known facts.
Give answer (A) if only conclusion I follows; (B) if only conclusion II follows; (C) if either I or II follows (D) if neither I or II follows and (E) if both I and II. 
26. Statements :   All planets are moons.
                          All moons are stars.
      Conclusions : I. All moons are planets
                          II. All planets are stars
     Ans: B
 
27. Statements :  All men are dogs.
                          All dogs are cats.
     Conclusions : I. All men are cats
                          II. All cats are men
     Ans: A
28. Statements : All tubes are handles
                         All cups are handles
      Conclusions : I. All cups are tubes
                           II. Some handles are not cups
    Ans: D
29.Statements :  All bags are cakes
                         All lamps are cakes
     Conclusions : I. Some lamps are bags
                          II. No lamp is bag.
    Ans: D
30.Statements : All flowers are stems
                        All stems are roots
     Conclusions : I. All roots are flowers
                         II. All stems are flowers
    Ans: D
3.  English      (30 quest)
Directions for Questions 1 to5: The passage given below is followed by a set of five questions. Choose the most appropriate answer to each question.
Language is not a cultural artifact that we learn the way we learn to tell time or how the federal government works. Instead, it is a distinct piece of the biological makeup of our brains. Language is a complex, specialized skill, which develops in the child spontaneously, without conscious effort or formal instruction, is deployed without awareness of its underlying logic, is qualitatively the same in every individual, and is distinct from more general abilities to process information or behave intelligently. For these reasons some cognitive scientists have described language as a psychological faculty, a mental organ, a neural system, and a computational module. But I prefer the admittedly quaint term “instinct.” It conveys the idea that people know how to talk in more or less the sense that spiders know how to spin webs. Web-spinning was not invented by some unsung spider genius and does not depend on having had the right education or on having an aptitude for architecture or the construction trades. Rather, spiders spin spider webs because they have spider brains, which give them the urge to spin and the competence to succeed. Although there are differences between webs and words, I will encourage you to see language in this way, for it helps to make sense of the phenomena we will explore.

Thinking of language as an instinct inverts the popular wisdom, especially as it has been passed down in the canon of the humanities and social sciences. Language is no more a cultural invention than is upright posture. It is not a manifestation of a general capacity to use – symbols: a three year old, we shall see, is a grammatical genius, but is quite incompetent at the visual arts, religious iconography, traffic signs, and the other staples of the semiotics curriculum. Though language is a magnificent ability unique to Homo sapiens among living species, it does not call for sequestering the study of humans from the domain of biology, for a magnificent ability unique to a particular living species is far from unique in the animal kingdom. Some kinds of bats home in on flying insects using Doppler sonar. Some kinds of migratory birds navigate thousands of miles by calibrating the positions of the constellations against the time of day and year. In nature’s talent show we are simply a species of primate with our own act, a knack for communicating information about who did what to whom by modulating the sounds we make when we exhale.

Once you begin to look at language not as the ineffable essence of human uniqueness but as a biological adoption to communicate information, it is no longer as tempting to see language as an insidious shaper of thought, and, we shall see, it is not Moreover, seeing language as one of nature’s engineering marvels – an organ with “that perfection of structure and co-adaption which justly excites our admiration, ”in Darwin’s words- give us a new respect for your ordinary Joe and the much-maligned English language (or any language) The complexity of language, from the scientist’s point of view, is part of our biological birth right; it is not something that parents teach their children or something that must be elaborated in school – as Oscar Wilde said,” Education is an admirable thing, but it is well to remember from time to time that nothing that is worth knowing can be taught “pre scholar ‘stacit knowledge of grammar is more sophisticated than the thickest style manual or the most state-of-the-art computer language system, and the same applies to all healthy human beings, even the notorious syntax-fracturing professional athlete and the, you know, like, inarticulate teenage skateboarder. Finally, since language is the product of a well-engineered biological instinct, we shall see that it is not nutty barrel of monkeys that entertainer-columnists make it out to be.
1. According to the passage, which of the following does not stem from popular wisdom on language?
A.  Language is a cultural artifact.
B.  Language is a cultural invention.
C.  Language is learnt as we grow.
D.  Language is unique to Homo sapiens.
E.  Language is a psychological faculty.
          Ans: E

2. Which of the following can be used to replace the “spiders know how to spin webs” analogy as used by the author?
A.  A kitten learning to jump over a wall
B.  Bees collecting nectar.
C. A donkey carrying a load.
D. A horse running a Derby.
E A pet dog protecting its owner’s property.
Ans: B
3. According to the passage, which of the following is unique to human beings?
A.  Ability to use symbols while communicating with one another.
B.  Ability to communicate with each other through voice modulation.
C.  Ability to communicate information to other members of the species.
D.  Ability to use sound as means of communication.
E.  All of the above.
Ans: B
4. According to the passage, complexity of language cannot be taught by parents or at school to children because
A.  children instinctively know language.
B.  children learn the language on their own.
C.  language is not amenable to teaching.
D. children know language better than their teachers or parents.
E. children are born with the knowledge of semiotics.
Ans: A
5. Which of the following best summarizes the passage?
A.  Language is unique to Homo sapiens.
B.  Language is neither learnt nor taught.
C.  Language is not a cultural invention or artifacts as it is made out.
D.  Language is instinctive ability of human beings.
E.  Language is use of symbols unique to human beings.
 Ans: D
Directions 6-10 : In each of the following questions, there is a certain relationship between two given words on one side of : : and one word is given on another side of : :while another word is to be found from the given alternatives, having the same relation with this word as the words of the given pair bear. Choose the correct alternative.
6. Forecast : Future : : Regret 😕
(A) Present     (B) Atone     (C) Past     (D) Sins
Ans: (C)
7. Fear : Threat : : Anger : ?
 (A) Compulsion     (B) Panic     (C) Provocation     (D) Force
Ans: (C)
9. Sculptor : Statue : : Poet : ?
(A) Canvas     (B) Pen     (C) Verse     (D) Chisel
Ans: (C)
10. Paw : Cat : : Hoof : ?
(A) Horse     (B) Lion     (C) Lamb     (D) Elephant
Ans: (A)
Directions 11-15: In each of the following questions, out of the given alternatives, choose the one which       best expresses the meaning of the given word.

11. Difficult
(A) retractable     (B) awe-inspiring     (C) lustrous     (D) formidable
Ans: (A)
12. Understatement
(A) distortion     (B) estimation     (C) animation     (D) pretentiousness
Ans: (A)
13. Outlandish
(A) foreshadowing     (B) inclusive     (C) remote     (D) omnipresent
Ans: (A)
14. Rare
 (A) malicious     (B) abstruse    (C) relentless     (D) overpowering
Ans: (B)
15. Felicity
(A) Anger    (B) Millionaire    (C) Bliss    (D) Feathery
Ans: (C)
Directions 16 to 20: Pick out from the words, given below each sentence, the  word which would  complete the sentence correctly and meaningfully.            
16. I will have to wait on this side of the road until the policeman…………….. the traffic.
    A) will stop    B) would stop    C) could stop    D) stops
    Ans: D
17. He went away before I………….. him the whole story.
    A) can tell    B) could tell    C) would have told    D) can told
    Ans: B
18. He could never have known the secret unless you……………….him.
    A) had told it to    B) tell it to    C) told it to    D) will tell it to
    Ans: A
19. It was one of …………………biggest mistakes that I have ever made.
    A) a    B) that    C) the     D) my
    Ans: C
20. She was one of the nicest persons……………taught us.
    A) whoever    B) that ever    C) ever    D) whomever
    Ans: A
Directions for Questions 21 to25 :Each of the following sentences has been divided into four parts.  There is an error in one of the parts. Point out the part which has an error 
21. A) We cannot have peace until /
     B) all the nations dealt with each other /
    C) in a spirit of equality and friendliness /
    D) and until we develop a new spirit of special integration.
     Ans: A      
22. A) That Christ is the embodiment of God on earth,/
    B) and that he is the Son of God, /
    C) are the belief /
    D) of all orthodox Christians even today.
    Ans: C
23. A) The thief, when he was passing through the gallery, /
    B) was seen by the servant /
    C) who fired him /
    D) with a pistol in his hand
       Ans: C
24. A) If my uncle will come today, /
    B) I shall leave for Calcutta with him./
    C) and stay therefore more than three months / 
    D) and learn Bengali.
        Ans: A
25. A) I laboured very hard / 
      B) lest I may fail
      C) or lose my division /
      D) at the very end of my educational career.
       Ans: A
26.A. 1971 war changed the  political geography of the subcontinent
B. Despite the significance of the event . there has been no serious book about the conflict
C. Surrender at Dacca aims to fill this gap
D. It also profoundly altered the geo-strategic situation in South-East Asia

1] ACBD   2] CADB   3] BADC   4] ADBC  

Ans: 4
27.A. Thus begins the search for relief: painkillers, ice, yoga, herbs, even surgery
B. Most computer users develop disorders because they ignore warnings like tingling fingers, a numb hand or a sore shoulder
C. They keep pointing and dragging until tendons chafe and scar tissue forms, along with bad habits that are almost impossible to chage
D. But cures are elusive , because repetetive stree injuries present a bag of ills that often defy easy disgnosis.

1] BDAC   2] BADC   3] BCAD   4] ABCD

Ans: 3
28.A. If you are used to having your stimulation come in from outside, your mind never develops its own habits of thinking and reflecting
B. Marx thought that religion was the opiate, because it soothed people’s pain and suffering and prevented them from rising in rebellion
C. If Karl Marx was alive today, he would say that television is the opiate of the people.
D. Television and similar entertainments are even more of an opiate because of their addictive tendencies.

1] BACD   2] ADBC   3] BDCA    4] CBDA

Ans: 4
29.A. Then two astronomers—the German, Johannes Kepler, and the Italian, Galileo Galilei—started publicly to support the Copernican theory, despite the fact that the orbits it predicted did not quite match the ones observed.
B. His idea was that the sun was stationary at the centre and that the earth and the planets move in circular orbits around the sun.
C. A simple model was proposed in 1514 by a Polish priest, Nicholas Copernicus.
D. Nearly a century passed before this idea was taken seriously.

1] CDBA   2] CBDA   3] BCAD   4] CADB

Ans:2
30.A. By the time he got to Linjeflug four years later, he had learned many lessons, in fact, he began his second stint as top dog by calling the entire company together in a hanger and asking for help, a far cry from his barking out commands just 48 months back.
B. At SAS, he arrived at a time crisis.
C. This book is chock-a-block full of intrusive stories and practical advice, describing Carton’s activities at Vingresor (where he assumed his first presidency at age 32), Linjeflug, and SAS in particular.
D. He began at Vingresor as an order giver, not a listener – neither to his people nor to his customers and made every mistake in the book.

1] BADC   2] BACD   3] CBAD   4] CDAB

Ans:4 

Total 90 questions , 90 minutes  —SECTIONAL CUTOFF IS THERE
1.  Aptitude     (30 quest)
2.  Reasoning (30 quest)
3.  English      (30 quest)
1.  Aptitude     (30 quest)
1. From a vessel, 1/3rd of the liquid evaporates on the first day. On the second day 3/4th of the remaining liquid 
    evaporates. What fraction of the volume is present at the end of the second day.
    Ans: 50%.
2. An orange glass has orange juice and white glass has apple juice both of equal volumes. 50 ml of the orange juice is 
     taken and poured into the apple juice. 50 ml from the white glass is poured into the orange glass. Of the two 
     quantities, the amount of apple juice in the orange glass and the amount of orange juice in the white glass, which one 
      is greater and by how much?
    Ans: The two quantities are equal.
3. There is a 4 inch cube painted on all sides. This is cut down into of 1 inch cubes. What is the no of cubes which 
    have no pointed sides?  
    Ans: 8.
4. Sam and Mala have a conversation. Sam says I am certainly not over 40. Mala says I am 38 and you are               
    at least 5 years older than me. Now, Sam says you are atleast 39. All the statements by the two are false.           
    How old are they really?
    Ans: Mala = 38 yrs; Sam = 41 yrs.
5. There is a certain four digit number whose fourth digit is twice the first digit. Third digit is three more than            
    second digit. Sum of the first and fourth digits twice the third number. What was that number?
    Ans: 2034 and 4368.
6. In a railway station, there are two trains going. One in the harbour line and one in the main line, each having a 
   frequency of 10 minutes. The main line service starts at 5 o’clock and the harbour line starts at 5.02A.M. A man 
   goes to the station every day to catch the first train that comes. What is the probability of the man catching the      
   first train?
    Ans: 0.8.
7. A family X went for a vacation. Unfortunately it rained for 13 days when they were there. But whenever it rained     
    in the mornings, they had clear afternoons and vice versa. In all they enjoyed 11 mornings and 12 afternoons.     
   How many days did they stay there totally?
   Ans: 18.
8. There is a safe with a 5 digit No. The 4th digit is 4 greater thansecond digit, while 3rd digit is 3 less than 2nd digit. 
    The 1st digit is thrice the last digit. There are 3 pairs whose sum is 11. Find the number.
    Ans: 65292
9. Bird is flying 120 km/hr between B to R. Two trains at B to R at 60 kmph. The distance traveled by the bird    
    before it is killed?
     Ans: 120
10. Four persons have to cross the bridge they are having one torch light. Four persons take 1,2,5,10 minutes 
      respectively, when two persons are going the will take the time of the slowest person what is the time taken to 
      cross all of them.
     Ans: 17 minutes.
11 A vender solds two things at same cost 12 Rs with one item at 25% profit and other at 20% loss, by this 
     transaction he made profit or loss by how much?
    Ans: Loss,60%.
12. Two friends A and B are running up hill and then to get down. Length of road is 440 yards. A on his return journey 
       met B going up at 20 yards from top. If A has finished the race 5 minutes earlier than B, then how much time A 
       had taken to complete the race?.
       Ans: 6.3 minutes
13. In the month of October in a year has exactly four Mondays and four Fridays, find what day of week will be on the 
      20th of November of that year.
    Ans: 20th November was a wednesday.
14.Six persons A,B,C,D,E & F went to solider cinema. There are six consecutive seats. A sits in one of the seats 
     followed by B, followed by C and soon. If a taken one of the six seats , then B should sit adjacent to A. C      
     should sit adjacent A or B. D should sit adjacent to A, B,or C and soon. How many possibilities are there?
    Ans: 32 ways.
15. 4 mathematician has x apples. If he arranges them in rows of 3 one will be left. The same is the case with            
       5,7,9 apples. But when he arranged them in rows of 11, non will be left. Find the no. of apples.
     Ans: 946.
16. A merchant in the last day sells 2 lamps for Rs.12 price. He finds that he has got 25 % gain on one and 20%       
       lost on the other. Did he loose or gain overall? If so how much?  
        Ans: 60 paise Loss.
17. Mr. X got Paramvir Chakra in 1971. He died when his age was 1/59th the year of birth. What was his date of 
      birth?
      Ans: 1947.
18.  There are 6561 number of balls in a bag. Out of which one is heavy ball. In how many minimum number of 
       weighing you can find the heavy ball.
        Ans: 8.
19. The profit made by a company in one year is enough to give 6% return on all shares. But as the preferred         
     shares get on return of 7.5%, so the ordinary shares got on return of 5%. If the value of preferd shares is              
     Rs 4,000000, then what is the value of ordinary shares?
     Ans: Rs. 6,000000.
20. A & B two places. C & D are two people. C started from A and D started from B. When they meet each       
      other in the way C traveled 18 m more than D. Then C takes 13 and half a minute and D takes 24 minutes to 
      reach the other end. What was the distance between A & B.
      Ans: 126.]
21. A family I know has several children. Each boy in this family has as many sisters as brothers but each girl has  
      twice as many brothers as sisters. How many brothers and sisters are there?
      Ans: 4 boys and 3 girls
22. There is a 5 digit no. 3 pairs of sum is eleven each. Last digit is 3 times the first one. Third digit is 3 less than        
       the second. 4th digit is 4 more than the second one. Find the digit.
       Ans: 25296.
23. There are five thieves, each loot a bakery one after the other such that the first one takes 1/2 of the total no. of       
      the breads plus 1/2 of a bread. Similarly 2nd, 3rd, 4th and 5th also did the same. After the fifth one, no. of          
      breads remained are 3. Initially how many breads were there?
      Ans: 31.
24. A software engineer just returned from US, has eaten too much fat & put a lot of weight. Every Sunday he starts 
      walking 4 km/hr on level ground, then up at 3 kmhr, then back down hill at 6kmhr, then again on level ground at 
       4kmhr till he reaches his destination. If he returned home at 9 p.m., what distance did he covered?
       Ans: 24 km
25. There are 4 married couples, out of which, 3 poeple in a group is needed. But there should not be his or her    
       spouse in the group. How many groups are possible?
        Ans: 32.
26. In the 4 digits 1,2,3,4, how many 4 digited numbers are possible which are divisible by 4? Repetitions are allowed.
        Ans: 64.
27. Two men are going along a track of rail in the opposite direction. One goods train crossed the first person in       
      20 sec. After 10 min the train crossed the other person who is coming in opposite direction in 18 sec.              
      After the train has passed, when the two persons will meet?
      Ans: Approx. 72 min,
28. At six o’clock the wall clock struck 6 times. Checking with my Watch, I noticed that the time between the first & 
      last strokes was 30 Seconds. How long will the clock take to stike 12 at mid night?
      Ans: 66 Seconds.
29. A person spending out 1/3 for cloths, 1/5 of the remaining for food and 1/4 of the remaining for travel. He is left 
      with Rs 100/-. How he had in the beginning?
        Ans: Rs 250/-.
30. A chain is broken into three pieces of equal lengths, containing 3 links each. It is taken to a blacksmith to join into  
      a single continuous one. How many minimum numbers of links are to to be opened to make it?
        Ans: 2.
2.  Reasoning (30 quest)
Answer Questions 1 to 5 on the basis of the information given below:
Nine  individuals – Z, Y, X, W, V, U, T, S and R – are the only candidates, who can serve on three committees– A, B and C, and each candidate should serve on exactly one of the committees.
committee: A should consist of exactly one member more than committee B.
                  It is possible that there are no members of committee C.
                  Among Z, Y and X none can serve on committee A.
                  Among W, V and U none can serve on committee G.
                  Among T, S and R none can serve on committee C.
1. In case T and Z are the individuals serving on committee B, how many of the nine individuals should serve on committee C?
A.3
B.4
C.5
D.6
E.7
Ans : B
2. Of the nine individuals, the largest number that can serve together on committee C is
A.9
B.8
C.7
D.6
E.5
Ans : D
3. In case R is the only individual serving on committee B, which among the following should serve on committee A?
A.W and S
B.V and U
C.V and T
D.U and S
E.T and S
Ans : E
4. In case any of the nine individuals serves on committee C, which among the following should be the candidate to serve on committee A?
A.Z
B.Y
C.W
D.T
E.S
Ans : C
5. In case T, S and X are the only individuals serving on committee B, the total membership of committee C should be:
A.Z and Y
B.Z and W
C.Y and V
D.Y and U
E.X and V
Ans : A
6. I am less than 6 feet tall but more than 2 feet tall. My height in inches is a multiple of 7 and is also 2 inches        
      more than a multiple of 6. What is my height in inches?
A. 43   B. 56   C. 21   D. 42   E. 65
Ans: B
7. A group of 12 girl scouts had enough food to last for 8 days when they arrived in camp. However, 4 more scouts 
    joined them without the amount of food being increased. How long will the food last if each scout is given the same 
   daily ration as originally planned?
A. 2     B. 3     C. 4     D. 5      E. 6
Ans: E
8. A dog takes 3 steps to walk the same distance for which a cat takes 4 steps. Suppose 1 step of the dog covers 1 
    foot. How many feet would the cat cover in taking 12 steps?
A. 9ft.     B. 3ft     C. 6ft     D.8ft      E. 16ft
Ans: A
9. If a kindergarten teacher places her children 4 on each bench, there will be 3 children who will not have a place. 
    However, if 5 children are placed on each bench, there will be 2 empty places. What is the smallest number of 
    children the class could have?
A. 33   B. 23   C. 21   D. 24   E. 20
Ans:  B
10. The distance around the track is 1000 yards. Each cyclist travels around the track at a different rate per minute. 
      For example, Cyclist A travels at 700 yards per minute. At time = 0, Cyclist A is at point 0 (or starting point). 
     After one minute, they would have traveled 700 yards. After two minutes, they would have traveled 1400 yards, 
     but because the track is circular, they would go around again, thus ending them 400 yards after the starting point. 
     After three minutes, adding 700 again, they would end up 100 yards after the starting point (because they went 
     around again). Continue this pattern for all three rates until all three cyclists are at the same point.
A.10mins   B. 11mins   C. 21mins   D. 9mins   E. 20mins
Ans: A
11. If SAVOURY is coded as OVUARSY then how will RADIATE be coded?
(A) AIDARET
(B) IDARA TE
(C) ARIADTE
(D) IDAATRE
(E) None of these
Ans : (D)
12. If MAPLE is coded as VOKZN then how will CAMEL be coded?
(A) OVNZF
(B) OUNZX
(C) OVNZX
(D) XZNVO
(E) None of these
Ans : (C)
13. If CRY is coded as MRYC then how will GET be coded?
(A) MTEG
(B) MGET
(C) MEGT
(D) METG
(E) None of these
Ans : (D)
14. If BURNER is coded as CASOIS then how will ALIMENT be coded?
(A) BKJLFMU
(B) EKOLIMS
(C) EMONIOU
(D) BRJSFTU
(E) EROSITU
Ans : (C)
15. If Sand is coded as Brick, Brick as House, House as Temple, Temple as Palace then where do you worship?
(A) Palace
(B) Temple
(C) Brick
(D) House
(E) None of these
Ans : (A)
16. How many such letter-pairs are there in the word SERVANT having the same no. of letters left between them in 
      the word as they have in the series?
(A) 2
(B) 3
(C) 4
(D) 5
(E) None of these
Ans : (A)
17. How many such letter-pairs are there in the word MONKEY having same no. of letters left between them as they 
      have in the series?
(A) 3
(B) 4
(C) 1
(D) 5
(E) None of these
Ans : (C)
18. How many such letter-pairs are there in the word SMUGGLER having same no. of letters left between them as 
      they have in the series?
(A) 2
(B) 3
(C) 4
(D) 1
(E) None of these
Ans : (A)
19. How many such letter-pairs are there in the word BONAFIDE having same no. of letters left between them as 
       they have in the series?
(A) 2
(B) 3
(C) 4
(D) 1
(E) None of these
Ans : (E)
20. How many such letter-pairs are there in the word FRONTIER having same no. of letters left between them as they have in the series?
(A) 2
(B) 4
(C) 1
(D) 3
(E) None of these
Ans : (A)
Directions 21-25 : In each question below are given two statements followed by two conclusions numbered I and II.  You have to take the given two statements to be true even if they seem to be at variance from commonly known facts. Read the conclusions and then decide which of the given conclusions logically follows from the two given statements, disregarding commonly known facts.
Give answer (A) if only conclusion I follows; (B) if only conclusion II follows; (C) if either I or II follows (D) if neither I or II follows and (E) if both I and II. 
21.Statements : Some soldiers are famous
                          Some soldiers are intelligent
    Conclusions : I. Some soldiers are either famous or intelligent
                          II. Some soldiers are neither famous nor intelligent
      Ans: D
22.Statements : All boys are honest
                           Sachin is honest
  Conclusions :   I. Sachin is a boy
                          II. All honest persons are boys
       Ans: D
23.Statements : Some nurses are nuns
                           Madhu is a nun
  Conclusions :  I. Some nuns are nurses
                          II. Some nurses are not nuns
      Ans: D
24.Statements : All windows are doors
                           No door is wall
  Conclusions : I. No window is wall
                         II. No wall is door
      Ans: A
25.Statements : All poles are guns
                          Some boats are not poles
     Conclusions : I. All guns are boats
                           II. Some boats are not guns
       Ans: D
26. How many such pairs of letters are there in the word GUARDIAN each of which has as many letters between 
       them in the word as in the English alphabet ?

(A) None
(B) One
(C) Two
(D) Three
(E) More than three
          Ans: D 

27. In a certain code language ‘pik da pa’ means ‘where are you’; ‘da na ja’ means ‘you may come’ and ‘na ka sa’ means ‘he may go’, which of the following means ‘come’ in that code language ?

(A) da
(B) ja
(C) na
(D) Cannot be determined
(E) None of these
        Ans: B
28. What should come next in the following number series ?   9 8 9 8 7 9 8 7 6 9 8 7 6 5 9 8 7 6 5 4 9 8 7 6 5
(A) 3
(B) 4
(C) 2
(D) 1
(E) None of these
      Ans: B
29. Meeta correctly remembers that her father’s birthday is after 8th July but before 12th July. Her brother correctly 
      remembers that their father’s birthday is after 10th July but before 15th July. On which day of July was definitely 
      their father’s birthday ?
(A) 10th
(B) 11th
(C) 10th or 11th
(D) Cannot be determined
(E) None of these
Ans: B
30. Four of the following five are alike in a certain way and so form a group. Which is the one that does not belong to that group ?
(A) B D F
(B) V X Z
(C) F I K
(D) M O Q
(E) L N P
        Ans: C
3.  English      (30 quest)
Directions for Questions 1 to 5: The passage given below is followed by a set of five questions. Choose the most appropriate answer to each question.
A remarkable aspect of art of the present century is the range of concepts and ideologies which it embodies. It is almost tempting to see a pattern emerging within the art field – or alternatively imposed upon it a posteriori – similar to that which exists under the umbrella of science where the general term covers a whole range of separate, though interconnecting, activities. Any parallelism is however – in this instance at least – misleading. A scientific discipline develops systematically once its bare tenets have been established, named and categorized as conventions. Many of the concepts of modern art, by contrast, have resulted from the almost accidental meetings of groups of talented individuals at certain times and certain places. The ideas generated by these chance meetings had two – fold consequences. Firstly, a corpus of work would be produced which, in great part, remains as a concrete record of the events. Secondly, the ideas would themselves be disseminated through many different channels of communication – seeds that often bore fruit in contexts far removed from their generation. Not all movements were exclusively concerned with innovation. Surrealism, for instance, claimed to embody a kind of insight which can be present in the art of any period. This claim has been generally accepted so that a sixteenth century painting by Spranger or a mysterious photograph by Atget can legitimately be discussed in surrealist terms. Briefly, then, the concepts of modern art are of many different (often fundamentally different) kinds and resulted from the exposures of painters, sculptors and thinkers to the more complex phenomena of the twentieth century, including our ever increasing knowledge of the thought and products of earlier centuries. Different groups of artists would collaborate in trying to make sense of rapidly changing world of visual and spiritual experience. We should hardly be surprised if no one group succeeded completely, but achievements, through relative, have been considerable. Landmarks have been established – concrete statements of position which give a pattern to a situation which could easily have degenerated into total chaos. Beyond this, new language tools have been created for those who follow – semantic systems which can provide a springboard for further explorations.

The codifying of art is often criticized. Certainly one can understand that artists are wary of being pigeon- holed since they are apt to think of themselves as individuals – sometimes with good reason. The notion of self-expression, however, no longer carries quite the weight it once did; objectivity has its defenders. There is good reason to accept the ideas codified by artists and critics, over the past sixty years or so, as having attained the status of independent existence – an independence which is not without its own value. This time factor is important here. As an art movement slips into temporal perspective, it ceases to be a living organism – becoming, rather, a fossil. This is not to say it becomes useless or uninteresting. Just as a scientist can reconstruct the life of a prehistoric environment from the messages codified into the structure of a fossil, so can an artist decipher whole webs of intellectual and creative possibility from the recorded structure of a ‘dead’ art movement. The artist can match the creative patterns crystallized into this structure against the potentials and possibilities of his own time. AS T.S Eliot observed, no one starts anything from scratch; however consciously you may try to live in the present, you are still involved with a nexus of behaviour patterns bequeathed from the past. The original and creative person is not someone who ignores these patterns, but someone who is able to translate and develop them so that they confirm more exactly to his – and our – present needs.
1. Many of the concepts of modern art have been the product of
(1) ideas generated from planned deliberations between artists, painters and thinkers.
(2) the dissemination of ideas through the state and its organizations.
(3) accidental interactions among people blessed with creative muse.
(4) patronage by the rich and powerful that supported art
(5) systematic investigation, codification and conventions.
      Ans. 3
 2. In the passage, the word ‘fossil’ can be interpreted as
(1) an art movement that has ceased to remain interesting or useful.
(2) an analogy from the physical world to indicate a historic art movement.
(3) an analogy from the physical world to indicate the barrenness of artistic creations in the past.
(4) an embedded codification of pre-historic life.
(5) an analogy from the physical world to indicate the passing of an era associated with an art movement.
Ans. 5
3.  In the passage, which of the following similarities between science and art may lead to erroneous conclusions?
(1) Both, in general, include a gamut of distinct but interconnecting activities.
(2) Both have movements not necessarily concerned with innovation.
(3) Both depend on collaborations between talented individuals.
(4) Both involve abstract thought and dissemination of ideas.
(5) Both reflect complex priorities of the modern world.
      Ans. 1

4. The range of concepts and ideologies embodied in the art of the twentieth century is explained by
(1) the existence of movements such as surrealism.
(2) landmarks which give a pattern to the art history of the twentieth century.
(3) new language tools which can be used for further explorations into new areas.
(4) the fast changing world of perceptual and transcendental understandings.
(5) the quick exchange of ideas and concepts enabled by efficient technology.
      Ans. 4
5.  The passage uses an observation by T.S. Eliot to imply that
(1) creative processes are not ‘original’ because they always borrow from the past.
(2) we always carry forward the legacy of the past.
(3) past behaviours and thought processes recreate themselves in the present and get labeled as ‘original’or’creative’.
(4) ‘originality’ can only thrive in a ‘green house’ insulated from the past biases.
(5) ‘innovations’ and ‘original thinking’ interpret and develop on past thoughts to suit contemporary needs.
      Ans. 5
Directions 6:10 Which of the phrases (A), (B), (C) and (D) given below each sentence should replace the phrases printed in bold in the following sentences to make the sentence grammatically correct. If the sentence is correct as it is and no correction is required, mark (E) as the answer.

6. Her face buries itself on his rough shirt and he could feel the fragrance of her hair and the warmth of her as she sobbed against his breast—
(A) sinking itself in his rough shirt
(B) buried itself in his rough shirt
(C) dipped itself in his rough shirt
(D) sank itself in his rough shirt
(E) No correction required
        Ans: B

7. ‘Allah’, he said, rising his face towards the star-spangled black sky, “punish me as much as you like-Mahesh died with thirst on his lips.”
(A) raised his face towards the star-spangled black sky
(B) had raised his face towards the star-spangled black sky
(C) rose his face towards the star spangled black sky
(D) raising his face towards the star-spangled black sky
(E) No correction required
Ans: D

8. All of us carried plenty of food and sweets with us and we served the little fortunate human beings with our own hands—
(A) The lesser fortunate human beings
(B) The less fortunate human beings
(C) The least fortunate human beings
(D) The unfortunate human beings
(E) No correction required
Ans: B

9. Once again, the Indian tradition of ‘ahimsa’ comes out as infinitely most relevant, than much of what we learn in modern education.
(A) The most relevant, than much of what we learn
(B) More relevance what we learn
(C) More relevant than much of what we learn
(D) No relevance in what we learn
(E) No correction required
Ans: C

10. When it was decided to send the rescue team in the colliery, the experts showed their reluctance.
(A) into the colliery
(B) inside the colliery
(C) under the colliery
(D) underneath the colliery
(E) No correction required
Ans: A
Directions(11-15): In each of the following questions, there is a certain relationship between two          
given words on one side of : : and one word is given on another side of : :while another  word is to be found from the given alternatives, having the same relation with this word as   the  words  of the given pair bear. Choose the correct alternative.        
11. Appraiser : Building : : Critic : ?
    (A) Book     (B) Masterpiece     (C) Judge     (D) Gold
    Ans: (A)
12. Tile : Mosaic : : Knot : ?
    (A) Embroidery     (B) Abacus     (C) Macrame     (D) Easle
    Ans: (C)
13. Bread : Yeast : : Curd : ?
    (A) Fungi     (B) Bacteria     (C) Germs     (D) Virus
    Ans: (B)
14. Command : Order : : Confusion : ?
    (A) Discipline     (B) Clarity     (C) Choas     (D) Problem
    Ans: (C)
15. Ruby : Red : : Sapphire : ?
    (A) Blue     (B) White     (C) Green     (D) Silver
    Ans: (A)
Directions 16 to 20: Pick out from the words, given below each sentence, the  word which would  complete the sentence correctly and meaningfully
16.Usually the ascent of a mountain face is much easier than the……………….
        A) fall    B) decent    C) descent    D) descend
        Ans: C
17. More than twenty years have now passed………………..I had my first flight.
    A) when   B) since    C) while    D) as
        Ans: B
18. The river overflowed its………………. and flooded the area
    A) Edges    B) Fronts    C) Limits    D) Banks
       Ans: D
19. Once you suspect a person of double dealing, you ought to keep him at arm’s.
    A) distance    B) length    C) aim    D) width
           Ans: B
20. While strolling on Janpath, I chanced to meet…………..European.
    A) one    B) the    C) an    D) a
           Ans: D
Directions for Questions 21 to25 :Each of the following sentences has been divided into four parts.  There is an error in one of the parts. Point out the part which has an error 

21. A) he has broken his pen, /
    B) and his paper is not yet complete, /
    C) but you can help him /
    D) if you give him your for an hour.
       Ans: D
22. A) This book is quite different /
    B) than the one I gave you, /
    C) because it is an old edition /
    D) and does not cover the whole course.
      Ans: B
23. A) He could come /
    B) if you invite him /
    C) well in time /
    D) in writing.
     Ans: B
24. A) Had, the doctor reached an hour earlier /
    B) and gave him proper medicine, /
    C) his life would have been saved /
    D) without any doubt.
      Ans: B
25. A) He ought have tried his best /
    B) from the very beginning /
    C) because the goal he set before him /
    D) was not an ordinary one.
      Ans: D
Directions 26-30 : In each of the following questions, out of the given alternatives, choose the one which is nearest in meaning to the given word.
26. ERROR
    A) Blunder    B) Nisadventure    C) Ambiguity    D) Misgiving
      Ans: A
27. ACRID
    A) Dirty    B) Pungent    C) Unripe    D) Bitter
      Ans: D
28. REQUITE
    A) Servile    B) Trashy    C) Ferocious    D) Juicy
      Ans: B
29. BADGER
    A) Attempt    B) Convince    C) Pester    D) Persuade
      Ans: C
30. CAUTIOUSLY
    A)Genuinely    B) Carefully    C) Secretly    D) Somewhat.
      Ans: B

WRITTEN TEST
Total 90 questions , 90 minutes  Offline (paper & pen) test—SECTIONAL CUTOFF IS THERE
1.  Aptitude     (30 quest)
2.  Reasoning (30 quest)
3.  English      (30 quest)
1.  Aptitude     (30 quest)
1. If he sells 40 mangoes, he will get the selling price of 4 mangoes extra, What is his % increase in profit ?
    Ans: 25% 
2. Two pipes can fill a tank in 10 and 12 hours respectively while third pipe will make the tank empty in 20 
     hours. If all three pipes operate simultaneously, in how many hours the tank will be filled ?

    Ans.7hours 30 minutes. 
3. Cost of an item is x. It’s value increases by p% and decreases by p% Now the new value is 1rupee,                        
    what is the actual value ? 

    Ans.(1000)/(1000-p*p). 
4. 6*12*15 is the volume of some material. How many cubes of edge 3 can be inserted into it ? 

     Ans.40 

5. 100 glasses are there. A servant has to supply glasses to a person If he supplies the glasses without                    
    any damage he will get 3 paise otherwise he will loose 3 paise. At the end of supplying 100 glasses if 
    he gets 270 paise, how many glasses were supplied safely.

    Ans.  95 
6. In a class composed of x girls and y boys what part of the class is composed of girls ?

     A. y/(x + y)
     B. x/xy
     C. x/(x + y)
     D. y/xy

     Ans. C

7. What is the maximum number of half-pint bottles of cream that can be filled with a 4-gallon can                           
    of cream(2 pt.=1 qt. and 4 qt.=1 gal)

        A.16
        B.24
        C.30
        D.64

        Ans.D

8. If the operation,^ is defined by the equation x ^ y = 2x + y, what is the value of a in 2 ^ a = a ^ 3

        A.   0
        B.   1
        C. -1
        D.  4

       Ans.B
9. A coffee shop blends 2 kinds of coffee, putting in 2 parts of a 33p. a gm. grade to 1 part of a 24p. a gm.            
    If the mixture is changed to 1 part of the 33p. a gm. to 2 parts of the less expensive grade, how much                
    will the shop save in blending 100 gms.

        A.Rs.90
        B.Rs.1.00
        C.Rs.3.00
        D.Rs.8.00

        Ans.C

10. There are 200 questions on a 3 hr examination. Among these questions are 50 mathematics problems.             
      It is suggested that twice as much time be spent on each maths problem as for each other question.                   
      How many minutes should be spent on mathematics problems

        A. 36
        B. 72
        C. 60
        D. 100

      Ans.B

11. In a group of 15,7 have studied Latin, 8 have studied Greek, and 3 have not studied either. How many of         
      these studied both Latin and Greek

        A.0
        B.3
        C.4
        D.5

      Ans.B

12. In June a baseball team that played 60 games had won 30% of its game played. After a phenomenal           
      winning streak this team raised its average to 50% .How many games must the team have won in a                  
      row to attain this average?
        A. 12
        B. 20
        C. 24
        D. 30
       Ans. C
13. M men agree to purchase a gift for Rs. D. If three men drop out how much more will each have to                         
      contribute towards the purchase of the gift ?
        A. D/(M-3)
        B. MD/3
        C. M/(D-3)
        D. 3D/(M2-3M)
        Ans. D
14. A company contracts to paint 3 houses. Mr. Brown can paint a house in 6 days while Mr. Black would             
      take 8 days and Mr. Blue 12 days. After 8 days Mr. Brown goes on vacation and Mr. Black begins to          
      work for a period of 6 days. How many days will it take Mr. Blue to complete the contract?
        A. 7
        B. 8
        C. 11
        D. 12

        Ans.C
15. 2 hours after a freight train leaves Delhi a passenger train leaves the same station traveling in the                             
      same direction at an average speed of 16 km/hr. After traveling 4 hrs the passenger train overtakes                 
      the freight train. The average speed of the freight train was?
        A. 30
        B. 40
        C.58
        D. 60
        Ans. B

16. A fold density is 19 times greater than the water and for copper it is 9 times. At what ratio you can mix gold      
      and copper to get 15 times denser than water. ?
        Ans: 3 : 2
17. There is a room with 6′ x 8′. A 1′ tile is fixed along the 4 walls in one row. How many 1″ tiles require to         
      finish the work. ?
          Ans: 24
18.  2 persons can finish a job in 8 days. First person alone can finish the work in 24 days. How many days          
       does the second person take to finish the job?
        Ans: 12 days

19. In a class total 34 students, 16 are have a brother, 15 are have sisters, 9 students don’t have either brothers     
      or sisters. Find the number of students having both brother and sisters.
        Ans: 6
21. In a car wheel, two spokes cover 15 degree. Then for the entire car, how many spokes are there?
        Ans: 24
20. What is the angle of degree suspended when two hands of clock showing the time 2:30?
       Ans: 105 degrees
21. The age difference between two brothers is 3 years. After 6 years the ratio between the age is 9:8. What are their ages?
        Ans: 21 and 18
22. A person’s salary is getting reduced by 20%. What percentage should be added to get back his original salary?
        Ans: 25%
23. Two persons start at the same point, walk in opposite directions with 5 km/hr and 5.5 km/hr respectively.          
      What is the distance separated after 2 and half hrs?
      Ans: 26.25 (approx)
24. A person starts walking at a speed of 5 km/hr through half the distance, rest of the distance he covers with     
      a speed 4km/hr. Total time of travel is 9 hours. What is the maximum distance he can cover?
      Ans: 40km
25. Initially two cups of same volume are present with milk filled upto 3/5th and 4/5th of their volumes.                
      Water is then filled. Then two mixtures are mixed. Find the ratio of water to milk in the mixture.
       Ans: 3 : 7
26. 16 grams of radioactive material decays into 8 grams in 10 years. How long will it take to decay to 1 gram?
      Ans: 70 yrs
27. In a rectangle the length is increased by of the original length. By what proportion should the width be reduced       
      so that the area will be the same?
     Ans: 33
28. If a square is formed by the diagonal of the square as an edge, what is the ratio between the area?
     Ans: 2
29. The perimeter of a rhombus is 52 units. One of its diagonal is 24 units. What is its second diagonals length?
    Ans: 10
30. A batsman scored 18 runs in his 18th innings and that makes his average 18. Find his average up to the 17th 
      innings?
    Ans: 19
2.  Reasoning (30 quest)
1. One of the following is my secret word: AIM DUE  MOD  OAT  TIE. With the list in front of you, if  I were     
    to tell you any one of my secret word, then you would be able to tell me the number of vowels in my secret word.    
   Which is my secret word?
     Ans. TIE
2. One of Mr. Horton, his wife, their son, and Mr. Horton’s mother is a doctor and another is a lawyer.
    a) If the doctor is younger than the lawyer, then the doctor and the lawyer are not blood relatives.
    b) If the doctor is a woman, then the doctor and the lawyer are blood relatives.
    c) If the lawyer is a man, then the doctor is a man.    Whose occupation you know?

   Ans. Mr. Horton: he is the doctor.
3. Three piles of chips–pile I consists one chip, pile II consists of chips, and pile III consists of three 
      chips are to be used in game played by Anita and Brinda. The game requires:
   a) That each player in turn take only one chip or all chips from just one pile.
   b) That the player who has to take the last chip loses.
   c) That Anita now have her turn.  From which pile should Anita draw in order to win?

     Ans. Pile II 
4. Mr. and Mrs. Aye and Mr. and Mrs. Bee competed in a chess tournament. Of the three games played:
      a) In only the first game were the two players married to each other.
      b) The men won two games and the women won one game.
      c) The Ayes won more games than the Bees.
      d) Anyone who lost game did not play the subsequent game. Who did not lose a game?

        Ans. Mrs.Bee 
5.  If A to Z is written in the reverse order which is the 5th letter to the right of the 11th letter from the left ?
        Ans . K

Directions 6-10  The office staff of XYZ corporation presently consists of three bookkeepers A, B, C and 5 secretaries D, E, F, G, H. The management is planning to open a new office in another city using 2 bookkeepers and 3 secretaries of the present staff . To do so they plan to separate certain individuals who don’t function well together. The following guidelines were established to set up the new office
I. Bookkeepers A and C are constantly finding fault with one another and should not be sent together to the new office as a team
II. C and E function well alone but not as a team , they should be separated
III. D and G have not been on speaking terms and shouldn’t go together
IV Since D and F have been competing for promotion they shouldn’t be a team
6. If A is to be moved as one of the bookkeepers, which of the following cannot be a possible working unit.

    A. ABDEH
    B. ABDGH
    C. ABEFH
    D. ABEGH

   Ans.B

7. If C and F are moved to the new office, how many combinations are possible

    A. 1
    B. 2
    C. 3
    D. 4

Ans.A

8. If C is sent to the new office, which member of the staff cannot go with C

    A. B
    B. D
    C. F
    D. G

   Ans.B

9. Under the guidelines developed, which of the following must go to the new office

    A. B
    B. D
    C. E
    D. G

   Ans.A

10. If D goes to the new office, which of the following is/are true

    I. C cannot go
    II. A cannot go
    III. H must also go

    A. I only
    B. II only
    C. I and II only
    D. I and III only

   Ans.D
11.If Leader is coded as Actor, Actor as an Administrator, Administrator as a Traitor, Traitor as a Social-reformer, 
     then who takes part in the Parliamentary elections?
    (A) Administrator
    (B) Leader
    (C) Traitor
    (D) Social-Reformist
    (E) None of these
    Ans : (E)
12.  If ‘245’ means—’Art and Talent’ in a certain code language, ‘316’ means— ‘Callous to Generous’, ‘147’ means— 
      ‘Callous and Polite’ then what is the code used for ‘to’ ?
    (A) Only 3
    (B) Only 1
    (C) 3 or 6
    (D) Only 6
    (E) None of these
    Ans : (C)
13. In a certain code language ‘579’ means—’Kanchan is soft-spoken’, ‘694’ means—‘Soft-spoken beautiful pure’, 
      ‘473’ means—‘Ganga is pure’, then what is the code used for ‘Kanchan’ ?
    (A) 7
    (B) 5
    (C) 9
    (D) Can not be determined
    (E) None of these
    Ans : (B)
14. Which of the following relates to FLOWER in the same way as RTERBN relates to SECTOR?
(A) RWLGPF       (B) EOFKUQ         (C) EOFMXS
(D) RWLEPD       (E) RWLEND
            Ans (E)
15. air is cloud, cloud is rain, rain is water, water is sand so what is cloud?
        Ans :sand

Directions ( questions 16 to 20 ): Read the following information carefully and answer the questions given below :
(1)A, B, C, D and E are five friends.
(2)B is elder to E, but not as tall as C.
(3)C is younger to A, and is taller to D and E.
(4)A is taller to D, But younger to E.
(5)D is elder to A but is shorter in the group.
16. Who among the following is the eldest?
       A). A     B). B      C). C     D). D     E). None of these
        Ans: E
17. Which of the following pairs of students is elder to D?
      A) BA     B). BC     C). BE     D) EA     E) None of these
      Ans: E
18. Which of the following statements is correct about B?
     1) B is not the tallest     2) B is shorter to E    
     (3)When they are asked to stand in ascending order with respect to their heights, B is in the middle
     A) Only (1) is correct    B) Only (1) and (3) are correct     C) All are correct
     D) All are incorrect     E) None of these
     Ans: A
19. If F, another friend is taller than C, how many of them will be between F and E according to their height?
     A) None     B) One    C) Two    D) Three     E) None of these
     Ans: E
20. If a selection is to be made among them who would be relatively older and also taller, who among them                 
      should be chosen?
     A) A     B) B    C) C     D) D     E) E
        Ans: B
Directions (questions 21 to 25): Each of the questions below consists of a question and two statements marked I and II given below it. You have to decide whether the data provided in the statements are sufficient to answer the question. Read both the statements and give answer
(A) If the data in statement I alone are sufficient to answer the question, while the data in statement II alone are not sufficient to answer the question. (B) If the data in statements I alone are not sufficient to answer the question, while  the data in statement II alone are sufficient to answer the question. (C) If the data either in statement I or in     statement II alone are sufficient to answer the question. (D) If the data even in both the statements I and II           together are not sufficient to answer the question. (E) If the data in both statements I and II together are needed to         answer the question.
21.. What does ‘Ne’ stands for in the code language?
     I. ‘Na Ni Nok Ne’ means ‘I will tell you’ and ‘Ni Nok Ne Nam’ means ‘he will tell you’ in that code language.
    II. ‘Ni Ne Mo Nam’ means ‘will he call you’ and ‘Ne Mok Sac Ni’ means ‘how will you go’ in that code language.
     Ans (D)
22. Who amongst P, Q, R, S, T and U is the tallest?
    I. P is taller that R and T but not as tall as U, who is taller than Q and S.
    II. R is the third in height in the ascending order and not as tall as U, P and Q, Q being taller than P but not the 
         tallest.
     Ans (C)
23. Who among A, B, C, D, E & F read the book last?
    I. F, who gave the book to B after reading, was third to read the same.
    II. C, who read the book after A, was the third person to read the book before it reached E.
    Ans (D)
24. Who is paternal uncle of Pavan?
    I. Pavan is brother of Poornima, who is daughter of Meena, who is sister of Kumar, who is brother Smrithi.
    II. Prithvi is brother of Indrajith, who is husband of Poornima, who is mother of Ganga, who is sister of Pavan.
   Ans (B)
25. What is Milan’s rank in the class of 44 students?
    I. Ramesh, whose rank is 17th in the class, is ahead of Shyam by 6 ranks, Shyam being 7 ranks ahead of Milan.
    II. Suketu is 26 ranks ahead of Milan and Shyamala is 6 ranks behind Milan while Savita stands exactly in the   
         middle of Shyamala and Suketu in ranks, her rankbeing 17.
      Ans (c)
Directions 26-27: Each question consists of five statements followed by options consisting of three       
                              statements put together in a specific order. Choose the option which indicates a        
                              valid argument, i.e. where the third statement is a conclusion drawn from the              
                              preceding two statements. 
26. A. All software companies employ knowledge workers.
      B. Tara Tech employs knowledge workers.
      C. Tara Tech is a software company.
      D. Some software companies employ knowledge workers.
      E. Tara Tech employs only knowledge workers.

       A) ABC       B) ACB         C) CDB            D) ACE
        Ans: B         
27. A. Traffic congestion increases carbon monoxide in the environment.
      B. Increase in carbon monoxide is hazardous to health.
      C. Traffic congestion is hazardous to health.
      D. Some traffic congestion does not cause increased carbon monoxide.
      E. Some traffic congestion is not hazardous to health.

        A. CBA          B. BDE          C. CDE          D. BAC
        Ans: D
28.  All scientists are fools. All fools are literates.

        A. All literates are scientists
        B. All scientists are literates
        C. No scientists are literates
        D. Both (a) and (b) are correct

        Ans. B

29.  All shares are debentures. No debentures are deposits.

         A.  All shares are deposits
         B.  Some shares are deposits
         C.  No shares are deposits
         D.  None of the above

         Ans. C

30. Some green are blue. No blue are white.

        A. No green are white
        B. Some green are white
        C. No green are white
        D. None of the above

        Ans. B

3.  English      (30 quest)  

Directions 1-5
To those who do listen, the desert speaks of things with an emphasis quite different from that of the shore, the mountain, the valley or the plains, whereas these invite action and suggest limitless opportunity and exhaust less resources, the implications and the mood of the desert are something different. For one thing , the desert is conservative not radical. It is more likely to provide awe than to invite conquest. The heroism which it encourages is the heroism of endurance, not that of conquest. It brings man up against this limitation, turns him upon himself and suggests values which more indulgent regions suppress. Sometimes it includes contemplation in men who have never contemplated before : And of all the answers to the question- what is a desert good for – ‘contemplation’ is perhaps the best.
1. In order to receive the desert’s message, the beholder needs to be
    a) courageous in his reaction.
    b) conservative in his responses.
    c) A good listener.
    d) Sensitive to nature.
   Ans 😀
2. The desert is unique among landscapes in that it encourages only
    a) contemplation
    b) Indolence
    c) Heroic Endeavor
    d) Adventurous Spirit
      Ans :A
3. If one responds with insight to the mood of the desert, it evokes
    a) An inclination for deep thought
    b) The possibility of unending resources
    c) The desire for Heroic conquest
    d) A sense of intense revulsion
      Ans :A
4. The writer calls the desert “conservative rather than radical” because it provides an environment that
    a) Inspires man to explore it
    b) Offers unlimited opportunity to conquer
    c) Tests ones Endurance
    d) makes one gloomy
      Ans :C
5. What does the phrase ”it brings man up against his limitations”, mean?
    a) It makes man feel hopeless about his limitations
     b) It makes man aware of his limitations
     c) It compels man to fight against his limitations
     d) It persuades man to overcome his limitations
    Ans 😀
Directions 6-10: In each of the following questions, there is a certain relationship between two given words on one side of : : and one word is given on another side of : :while another word is to be found from the given alternatives, having the same relation with this word as the words of the given pair bear. Choose the correct alternative.
  
6. Errata : Books : : flaws:?
    (A) Manuscripts     (B) Metals     (C) Speech     (D) Charter
    Ans: (B)
7. Planet : Orbit : : Projectile : ?
     (A) Trajectory     (B) Track     (C) Milky way     (D) Path
    Ans: (A)
8. Appraiser : Building : : Critic : ?
      (A) Book     (B) Masterpiece     (C) Judge     (D) Gold
     Ans: (A)
9. King : Throne : : Rider : ?
     (A) Seat     (B) Horse     (C) Saddle     (D) Chair
     Ans: (C)
10. Reluctant : Keen : : Remarkable : ?
     (A) Usual     (B) Restrained     (C) Striking     (D) Evolution
      Ans: (A)
Directions 11-15: In each of the following questions, out of the given alternatives, choose the one which       best expresses the meaning of the given word.
11. BEMOAN
       A. Lament    B. Soothe     C. Denounce     D. Loathe
        Ans: A
12. ABRADE
       A. Scold       B. attach        C. Wear off        D. Twist
       Ans: C
13. DISTINGUISH
       A. Darken    B. Abolish    C. Differentiate       D. Confuse
       Ans: A
14. PORTRAY
      A. Communicate       B. Paint        C. Express       D. Draw
      Ans: D
15. INFINITE
      A. Strange       B. Endless        C. Indefinite        D. Vague
      Ans: B
 
Directions 16-17: Read each sentence to find if there is any grammatical error in it. If there is any error,     it will be only one part of the sentence. The number or alphabet of that part is your answer.
16.  I shall / ring him / tomorrow / in the afternoon.
        A           B             C                 D
  
             Ans: B
17.  I enjoyed / during my / stay in / England.
         A               B             C           D
        
             Ans: A
18. I believe / that respect / is more preferable than / money.
         A                B                          C                          D
 
            Ans: C
19. I never have / visited / or intend to visit / foreign countries /
            A                B                 C                          D
 
           Ans: C
20. In this way nuclear fission / or the splitting / of the atom / have been achieved /
                      A                               B                    C                        D
 
           Ans: D
Directions 21 to 25: Pick out from the words, given below each sentence, the  word which would  complete the sentence correctly and meaningfully.
 
21. Rajeev failed in the examination because none of her answers were ____ to the questions asked
       a) allusive     b) revealing     c) pertinent     d) referential      e) impeccable
       Ans: D
22. There are ______ views on the issue of giving bonus to the employees
        a) independent     b) divergent     c) modest     d) adverse     e) valuable
        Ans: B
23. Man who has committed such an ____ crime must get the most severe punishment.
        a) injurious     b) uncharitable     c) unworthy     d) admoniable     e) irreproachable
       Ans: A
24. He has_____ people visiting him at his house because he fars it will cause discomfort to neighbors
        a) curtailed     b) requested     c) stopped    d) warned     e) forbidden
        Ans: C
25. Although he never learnt to read, his exceptional memory and enquiring mind eventually made him a very____ man.
       a) dedicated     b) erudite     c) pragmatic    d) benevolent     e) charismatic
        Ans: A
Directions for Questions 26-30: In each of the following questions, a paragraph or a sentence has been broken up into different parts. The parts have been scrambled and numbered as given below.  Choose the correct order of these parts from the given alternatives.
26. 1) To get home early     2) I phoned my mother     3) That I wouldn’t able    4) to tell her   
       5) as I had promised    6)When I reached my office
        A. 6, 1, 4, 2, 3, 5
        B. 6, 2, 4, 3, 1, 5
        C. 5, 2, 4, 3, 1, 6
        D. 6, 2, 1, 3, 4, 5
         Ans: B
27.  1) From the corporate sector     2) The department has initiated steps    3) the development of wastelands 
        4) to evolve appropriate schemes      5) and financial institutions for     6) for mobilizing investments  
        A. 1, 5, 3, 2, 4, 6, 
        B. 2, 3, 6, 1, 5,4
        C. 2, 4, 6, 1, 5, 3
        D. 2, 4, 6, 1, 3, 5
        Ans: C
28.  1) paintings    2) have been      3) totally ruined by rain   4)last night   5) which were submitted 6) for the exhibition
        A. 1, 5, 4, 2, 3,6
        B. 1, 4, 6, 2, 3,5
        C. 1, 5, 2, 6, 3,4
        D. 1, 5, 6, 2, 3,4
       Ans: D
29. 1) and also bills     2) to the office superintendent.      3) for the work done by him     4) the field worker
       5) at the end of the assignment     6) submitted his papers
       A. 5, 4, 6, 1, 3, 2
      B. 5, 4, 6, 1, 2, 3
      C. 5, 4, 1, 6, 3, 2
      D. 5, 6, 4, 1, 3, 2
       Ans: A
30.  1) Those who     2) expect others     3) cannot treat     4) should not     5) others well     6)to treat them well
        A) 1, 3, 5, 6, 2, 4
        B) 1, 5, 3, 4, 2, 6
        C) 1, 3, 5, 4, 2, 6
        D) 1, 3, 5, 4, 6, 2
         Ans: C

Total 90 questions , 90 minutes  —SECTIONAL CUTOFF IS THERE
1.  Aptitude     (30 quest)
2.  Reasoning (30 quest)
3.  English      (30 quest)
1.  Aptitude     (30 quest)
1. Three friends divided some bullets equally. After all of them shot 4 bullets the total number of bullets remaining is equal to the bullets each had after division. Find the original number divided. 
Ans: 18
2. In a class composed of x girls and y boys what part of the class is composed of girls

A.y/(x + y)
B.x/xy
C.x/(x + y)
D.y/xy

Ans.C

3 What is the maximum number of half-pint bottles of cream that can be filled with a 4-gallon can of cream(2 pt.=1 qt. and 4 qt.=1 gal)

A.16
B.24
C.30
D.64

Ans.D

4. If the operation,^ is defined by the equation x ^ y = 2x + y, what is the value of a in 2 ^ a = a ^ 3

A.0
B.1
C.-1
D.4

Ans.B

5. A coffee shop blends 2 kinds of coffee,putting in 2 parts of a 33p. a gm. grade to 1 part of a 24p. a gm.If the mixture is changed to 1 part of the 33p. a gm. to 2 parts of the less expensive grade,how much will the shop save in blending 100 gms.

A.Rs.90
B.Rs.1.00
C.Rs.3.00
D.Rs.8.00
Ans.C

6. There are 200 questions on a 3 hr examination.Among these questions are 50 mathematics problems.It is suggested that twice as much time be spent on each maths problem as for each other question.How many minutes should be spent on mathematics problems

A.36
B.72
C.60
D.100

Ans.B

7. In a group of 15,7 have studied Latin, 8 have studied Greek, and 3 have not studied either.How many of these studied both Latin and Greek

A.0
B.3
C.4
D.5

Ans.B
8. .If 13 = 13w/(1-w) ,then (2w)2 =

A.1/4
B.1/2
C.1
D.2

Ans.C
9. In june a baseball team that played 60 games had won 30% of its game played. After a phenomenal winning streak this team raised its average to 50% .How many games must the team have won in a row to attain this average?A. 12
B. 20
C. 24
D. 30
Ans. C
10. A company contracts to paint 3 houses. Mr.Brown can paint a house in 6 days while Mr.Black would take 8 days and Mr.Blue 12 days. After 8 days Mr.Brown goes on vacation and Mr. Black begins to work for a period of 6 days. How many days will it take Mr.Blue to complete the contract?
A. 7
B. 8
C. 11
D. 12

Ans.C
11. 2 hours after a freight train leaves Delhi a passenger train leaves the same station travelling in the same direction at an average speed of 16 km/hr. After travelling 4 hrs the passenger train overtakes the freight train. The average speed of the freight train was?A. 30
B. 40
C.58
D. 60
Ans. B
12  x,y and z are odd no. not necessarly in sequence.then y=?
    (1)mean of x and z is 114.
    (2)mean of y and z is 113.

        Ans: 115

13. ram is 27 year old then mohan. after 7 year ram age is thrice as mohan age .find ram age?
      Ans : 67/2

14  if three tapes are filling a tank of capacity of 500lit with speed of 30lit/sec,48lit/sec and 36lit/sec.                      
      Find after how long time tank will fill?
      Ans : 500/114
15. suppose in a pot 20 cards are there label 1-20 on that. find the probability of getting two prime.

        Ans : 8/20 = 2 / 5

16. If a person sells a product for rs141/- he suffers a loss of 6%.if he has to have a profit of 10%,
     at what price should he sell it?
    (a) rs250
    (b) rs.175
    (c) rs 195.
    (d) rs.165
    Ans.rs.165
17. A ball falls from a height of 8ft ,bounces back to half the distance & continues till it comes to rest.
     what is the total distance traveled by the ball?
     (a) 24ft
     (b)30 ft
     (c) infinite
     (d) cannot be determined
     Ans:(a)

18. If 3 houses are to be painted,mr A can paint a house in 6 days(nos are not same)…mr B can do the same in 8 
      days…& mr.C in 12 days.if mr A does the work for 8 days & leaves for vacation, & mr B continues the work for 
      the next 6 days, for how many days should mr.C work?
      Ans:11 days(check)
19. one student takes 20 mins to reach school if he goes at a speed of 15kmph .At what speed should he go to          
      reach in 15 mins ( nos are not same)
      Ans: 16kmph
20. How many rational numbers are there between 0 & 5 ?
       Ans:infinite
21. A cube of 3 unit has 6 surface is painted. If u cut the cube of 1 unit each how many cubes remain unpainted           
      on all sides ?
      Ans.1

22. one questions in profit and loss a person sold an article at Rs.141 at 6% loss, to gain a profit of 10% at what price 
       the article should be sold.
        Ans: 165 Rs
23. 1st day of the year 1999 is sunday and what will be last day of the same year
        Ans: sunday
24.  6 squares of equal size are placed side by side to form a rectangle whose perimeter is 182. find the perimeter of 
       the single square.
        Ans: 52

25 .The average age of students in a class is 16.when a teacher of age 40 is added , the average becoems 17. what is 
      the total number of students
      Ans.23
26.One persont travlled 4 laps with the speed of 10,20,40 ( or 30) and 60 kmph and what is the average speed.
        Ans: 20 
  27. If Rs20/- is available to pay for typing a research report & typist A  produces 42 pages and typist B produces   
        28 pages. How much should typist A  receive?
        Ans: :Rs12/-
28. 4 thieves rob a bakery of the bread one after the other. each thief takes half of what is present ,& half 
      a bread…if at the end 3 bread remains, what is the no of bread that was present initially?
      Ans:63
29. Six squares of same dimension are kept side by side to make a rectangle with a perimeter of 182cm.                      
       what is the perimeter of each square ?
       Ans: 52
29. one student takes 20 mins to reach school if he goes at a speed of 15kmph .At what speed should he go to           
      reach in 15 mins ( nos are not same)
      Ans: 16kmph( check)

30. A clerk multiplied a number by ten when it should have been divided by ten. The ans he got was 100.what should 
      the answer have been?
      Ans::1

2.  Reasoning (30 quest)
Answer Questions 1 to 5 on the basis of the information given below:
There are three on-off switches on a control panel A, B, and C. They have to be changed from an initial setting to a second setting according to the following conditions : In case only switch A is the switch on in the initial setting , then turn on switch B.
In case switches A and B are the only switches on in the initial setting, then turn on switch C. In case all the three switches are on initially setting, then turn off the switch C. For any other initial setting, turn on all switches that are off and turn off all switches, if any, that are on.
1. In case in the initial setting is the switches A and B are on and the switch C is off, then what could be the second setting?
A.A on, B on, C on.
B.A on, B off, C on.
C.A on, B off, C off.
D.A off, B on, C off.
E.A off, B off, C on.
Ans : A
2. In case switch B is the only switch on in the initial setting, what must be the second setting?
A.A on, B on, C on.
B.A on, B on, C off.
C.A on, B off, C on.
D.A off, B off, C on.
E.A off, B off, C off.
Ans : C
3. In case all the three switches are on in the second setting, which among the following could have been the initial setting ?
A.A on, B on, C on.
B.A on, B on, C off.
C.A on, B off, C on.
D.A on, B off, C off.
E.A off, B on, C off
Ans : B
4. In case switch A is off in the second setting, which among the following could have been the initial setting ?
A.A on, B on, C on.
B.A on, B on, C off.
C.A on, B off, C on.
D.A on, B off, C off.
E.A off, B on, C off.
Ans : C
5. In case only switch B is on in the second setting, which among the following could have been the initial setting ?
A.A on, B on, C on.
B.A on, B off, C on.
C.A off, B on, C off.
D.A off, B off, C on.
E.A off, B off, C off.
Ans: C
6. It was vacation time, and so I decided to visit my cousin’s home. What a grand time we had! In the mornings,       
    we both would go for a jog. The evenings were spent on the tennis court. Tiring as these activities were, we could  
    manage only one per day, i.e., either we went for a jog or played tennis each day. There were days when we felt 
    lazy and stayed home all day long. Now, there were 15 mornings when we did nothing, 9 evenings when we stayed 
    at home, and a total of 12 days when we jogged or played tennis. For how many days did I stay at my cousin’s 
    place?
    A. 18       B. 14        C. 15        D. 20
     Ans: A
7. If you were to construct a 7 × 7 checkered square (i.e., a 7 × 7 chess board), how many rectangles would there be 
    in total? You need to include squares too because a square is a special kind of rectangle.
     A. 784       B. 918        C. 842        D. 676          
     Ans: A
8. A block of wood in the form of a cuboid 4″ × 10″ × 13″ has all its six faces painted pink. If the wooden block is cut 
    into 520 cubes of 1″ × 1″ × 1″, how many of these would have pink paint on them?
A. 348       B. 352       C. 344        D. 340          
Ans: C
9. A tennis championship is played on a knock-out basis, i.e., a player is out of the tournament when he loses a match. 
    How many players participate in the tournament if 127 matches are totally played?
    A. 136       B. 144        C. 124        D. 128          
    Ans: C
10. A large water tank has two inlet pipes (a large one and a small one) and one outlet pipe. It takes 3 hours to fill the 
      tank with the large inlet pipe. On the other hand, it takes 4 hours to fill the tank with the small inlet pipe. The outlet 
      pipe allows the full tank to be emptied in 7 hours. What fraction of the tank (initially empty) will be filled in 1.70 
      hours if all three pipes are in operation?
       A. 0.75       B. 0.85        C. 0.80        D. 0.70
        Ans: A
Answer Questions 11 to 15 on the basis of the information given below:
K, L, M, N, P, Q, R, S, U and W are the only ten members in a department. There is a proposal to form a team from within the members of the department, subject to the following conditions:

A team must include exactly one among P, R, and S.
A team must include either M or Q, but not both.
If a team includes K, then it must also include L, and vice versa.
If a team includes one among S, U, and W, then it must also include the other two.
L and N cannot be members of the same team.
L and U cannot be members of the same team.

The size of a team is defined as the number of members in the team.

11.  What could be the size of a team that includes K?
    A. Only 2
    B. 3 or 4
    C. 2 or 3
    D. Only 4
    E. 2 or 4
    Ans: D
12. In how many ways a team can be constituted so that the team includes N?
        A. 5       B. 6    C. 4       D.  2       E. 3 
        Ans: B
13. What would be the size of the largest possible team?
        A. cannot be determined        B. 6        C. 7        D. 8        E. 5
        Ans: E
14. Who can be a member of a team of size 5?
        A. R        B.  L         C. P         D. M         E. K
        Ans: D
15. Who cannot be a member of a team of size 3?
          A. P        B.  L         C. M         D. N         E. Q
        Ans: B
16 On a certain day it took Bill three times as long to drive from home to work as it took Sue to drive from home to 
     work. How many kilometers did Bill drive from home to work?
    (1) Sue drove 10 kilometers from home to work, and the ratio of distance driven from home to work time to drive 
          from home to work was the same for Bill and Sue that day. 
    (2) The ratio of distance driven from home to work time to drive from home to work for Sue that  day was 64 
          kilometers per hour.
A. Both statements 1 and 2 together are sufficient to answer the question but neither statement is sufficient alone.
B. Statement 1 alone is sufficient but statement 2 alone is not sufficient to answer the question asked
C. Each statement alone is sufficient to answer the question.
D. Statements 1 and 2 are not sufficient to answer the question asked and additional data is needed to answer the statements
E. Statement 2 alone is sufficient but statement 1 alone is not sufficient to answer the question asked
Ans:B
17.  If x and y are nonzero integers, is x/y an integer? 
     (1) x is the product of 2 and some other integer. 
      (2) There is only one pair of positive integers whose product equals y
A. Statement 1 alone is sufficient but statement 2 alone is not sufficient to answer the question asked.
B. Statement 2 alone is sufficient but statement 1 alone is not sufficient to answer the question asked.
C. Each statement alone is sufficient to answer the question.
D. Both statements 1 and 2 together are sufficient to answer the question but neither statement is sufficient alone.
E. Statements 1 and 2 are not sufficient to answer the question asked and additional data is needed to answer the statements.
Ans: E
18  If n is an integer between 2 and 100 and if n is also the square of an integer, what is the value of n?
      (1) n is the cube of an integer. 
       (2) n is even
    A. Each statement alone is sufficient to answer the question.
    B. Statement 2 alone is sufficient but statement 1 alone is not sufficient to answer the question asked.
    C. Statement 1 alone is sufficient but statement 2 alone is not sufficient to answer the question asked.
    D. Both statements 1 and 2 together are sufficient to answer the question but neither statement is sufficient alone.
    E.  Statements 1 and 2 are not sufficient to answer the question asked and additional data is needed to answer     
         the statements.
        Ans: C
19. What is the average distance that automobile D travels on one full tank of gasoline? 
    (1) Automobile D averages 8.5 kilometers per liter of gasoline. 
    (2) The gasoline tank of automobile D holds exactly 40 liters of gasoline.
    A. Each statement alone is sufficient to answer the question.
    B. Statement 1 alone is sufficient but statement 2 alone is not sufficient to answer the question asked.
    C. Both statements 1 and 2 together are sufficient to answer the question but neither statement is sufficient alone.
    D. Statement 2 alone is sufficient but statement 1 alone is not sufficient to answer the question asked.
    E. Statements 1 and 2 are not sufficient to answer the question asked and additional data is needed to answer the    
        statements.
       Ans: C
20. What fraction of his salary did Mr.Johnson put into savings last week? 
    (1) Last week Mr. Johnson put $17 into savings. 
    (2) Last week Mr. Johnson put 5% of his salary into savings.
    Ans: D
Directions21- 25: In each of the questions from 1 to 5 a statement followed by two arguments is given. You have to decide which of the given statements is a strong argument and which is weak argument. Give answer (A) if only first argument is strong, give answer (B) if only second argument is strong, give answer (C) if either first or second argument is strong, give answer (D) if neither first nor second argument is strong and given answer (E) if both first and second arguments are strong.

21. Statement  :Should military training be made compulsory in our country like other countries?
      Arguments :    I. Yes, every citizen has right to protect his country.
                               II. No, it is against the policy of non-violence.
     Ans : (A)

22. Statement  : Should octroi be abolished?
      Arguments : I. No, it will adversely affect government revenues.
                           II. Yes, it will eliminate an important source of corruption.
      Ans : (E)

23. Statement   : Should competitive examinations for selecting candidates for jobs, be of objective test only?
      Arguments  :I. Yes, the assessment of objective test is reliable.
                           II. No, the number of questions to be answered is always very large.
      Ans : (A)

24. Statement  :  Should those who receive dowry be encouraged ?
      Arguments : I. Yes, due to this, the number of those who receive dowry will decrease.
                           II. No, due to this the number of those who receive dowry will increase.
      Ans : (B)

25. Statement  : Should jobs be delinked with academic degrees and diplomas?
      Arguments :I. Yes, a very large number of candidates will apply so that the competition will be tough and the 
                               candidates will prepare more.
                          II. No, importance of higher education will be diminished.
      Ans : (E)
26. If ‘678’ means—’Society Family Husbandry’, ‘574’ means—’Husbandry Health Control’, ‘342’ means—’Health   
      Census shop’, then—
     (I) Which code has been used for ‘Health’ ?
     (A) 7
     (B) 5
     (C) 4
     (D) 2
     (E) None of these
      Ans : (C)

27. (II) Which code has been used for ‘Census’ ?
        (A) 3 or 2
        (B) 3 or 7
        (C) 4
        (D) 5
        (E) 6
        Ans : (A)

28. (III) For which word code ‘6’ has been used?
        (A) Society
        (B) Family
        (C) Husbandry
        (D) Society or Family
        (E) None of these’
        Ans : (D)

29. If ‘KRN’ means—’Callous collission life’, ‘RTP’ means—’Life very sad’, ‘NPD’ means—’Collission sad future’ then 
      what is the code used for ‘Callous’ ?
        (A) R
        (B) N
        (C) K
        (D) Can not be determined
        (E) None of these
        Ans : (C)

30. If ‘MLT’ means—’Day is clear’, ‘LKS’ means—’Life is sad’, ‘SMMO’ means—’Clear or sad’, then what is the 
      code used for ‘Day’ ?
        (A) T
        (B) K
        (C) MO
        (D) L
        (E) None of these
        Ans : (A)
3.  English      (30 quest)  
Directions : In each of the following questions, four sentences are given. Choose the one which is grammatically correct.
1. A) wrote to him yesterday
    B) This is the road to go
    C) Let us aim to do good
    D) Take down his address in your copy
    Ans: A
2. The principal and Secretary are leave
    B) A number of students has failed in the examination
    C) It is I who has committed this crime
    D) many a man runs after money
    Ans: D
3. A) These three boys hate each other
    B) Sanjeev lent Rs.2000 to Vincent
    C) I, you and he belong to the same village
    D) Anyone of the two students can solve this sum
    Ans: B
4. A)He is a man whom I know is trustworthy
    B) Until you remain idle you will make no progress
    C) We talked about the weather
    D) Half dozen books were burnt
    Ans: C
5. A)The proceeds of the charity show are for riot victims.
    B) He asked Ajay and I to go
    C) The weather of this place does not suit me
    D) Either Rajesh or his friends has done it.
    Ans: A
Directions 6-10: In each of the following questions, there is a certain relationship between two given words on one side of : : and one word is given on another side of : :while another word is to be found from the given alternatives, having the same relation with this word as the words of the given pair bear. Choose the correct alternative.
6. Eye :Myopia : : Teeth : ?
(A) Pyorrhea     (B) Cataract     (C) Trachoma     (D) Eczema
Ans: (A)
7. Scribble : Write : : Stammer : ?
(A) Walk     (B) Play     (C) Speak     (D) Dance
Ans: (C)
8. Errata : Books : : flaws:?
(A) Manuscripts     (B) Metals     (C) Speech     (D) Charter
Ans: (B)
9. Breeze : Cyclone : : Drizzle : ?
(A) earth quake     (B) Storm     (C) Flood     (D) Down pour
Ans: (D)
10. Venerate : Worship : : Extol : ?
(A) Glorify     (B) Homage     (C) Compliment     (D) Recommend
Ans: (A)
Directions for Questions 11 to 15 The passage given below is followed by a set of five questions. Choose the most appropriate answer to each question.
When I was little, children were bought two kinds of ice cream, sold from those white wagons with the canopies made of silvery metal: either the two-cent cone or the four-cent ice cream pie. The two-cent cone was very small, in fact it could fit comfortably into a child’s hand, and it was made by taking the ice cream from its container with a special scoop and piling it on the cone. Granny always suggested I eat only a part of the cone, then throw away the pointed end, because it had been touched by the vendor’s hand (though that was the best part, nice and crunchy, and it was regularly eaten in secret, after a pretense of discarding it).

The four-cent pie was made by a special little machine, also silvery, which pressed two disks of sweet biscuit against a cylindrical section of ice cream. First you had to thrust your tongue into the gap between the biscuits until it touched the central nucleus of ice cream; then, gradually, you ate the whole thing, the biscuit surfaces softening as they became soaked in creamy nectar. Granny had no advice to give here: in theory the pies had been touched only by the machine; in practice, the vendor had held them against his hand while giving them to us, but it was impossible to isolate the contaminated area.

I was fascinated, however, by some of my peers, whose parents bought them not a four-cent pie but two two-cent cones. These privileged children advanced proudly with one cone in their right hand and one in their left; and expertly moving their head from side to side, they licked first one, then the other. This liturgy seemed to me so sumptuously enviable, that many times I asked to be allowed to celebrate it. In vain. My elders were inflexible: a four-cent ice, yes; but two two-cent ones, absolutely no.

As anyone can see, neither mathematics nor economy nor dietetics justified this refusal. Nor did hygiene, assuming that in due course the tips of both cones were discarded. The pathetic, and obviously mendacious, justification was that a boy concerned with turning his eyes from one cone to the other was more inclined to stumble over stones, steps, or cracks in the pavement. I dimly sensed that there was another secret justification, cruelly pedagogical, but I was unable to grasp it.

Today, citizen and victim of a consumer society, a civilization of excess and waste (which the society of the thirties was not), I realize that those dear and now departed elders were right. Two two-cent cones instead of one at four cents did not signify squandering, economically speaking, but symbolically they surely did. It was for this precise reason, that I yearned for them: because two ice creams suggested excess. And this was precisely why they were denied me: because they looked indecent, an insult to poverty, a display of fictitious privilege, a boast of wealth. Only spoiled children ate two cones at once, those children who in fairy tales were rightly punished, as Pinocchio was when he rejected the skin and the stalk. And parents who encouraged this weakness, appropriate to little parvenus, were bringing up their children in the foolish theater of “I’d like to but I can’t.” They were preparing them to turn up at tourist-class cheek-in with a fake Gucci bag bought from a street peddler on the beach at Rimini.

Nowadays the moralist risks seeming at odds with morality, in a world where the consumer civilization now wants even adults to be spoiled, and promises them always something more, from the wristwatch in the box of detergent to the bonus bangle sheathed, with the magazine it accompanies, in a plastic envelope. Like the parents of those ambidextrous gluttons I so envied, the consumer civilization pretends to give more, but actually gives, for four cents, what is worth four cents. You will throw away the old transistor radio to purchase the new one, that boasts an alarm clock as well, but some inexplicable defect in the mechanism will guarantee that the radio lasts only a year. The new cheap car will have leather seats, double side mirrors adjustable from inside, and a paneled dashboard, but it will not last nearly so long as the glorious old Fiat 500, which, even when it broke down, could be started again with a kick.

The morality of the old days made Spartans of us all, while today’s morality wants all of us to be Sybarites.
11. Which of the following cannot be inferred from the passage?
A.  Today’s society is more extravagant than the society of the 1930s.
B.  The act of eating two ice cream cones in akin to a ceremonial process.
C.  Elders rightly suggested that a boy turning eyes from one cone to the other was more likely to fall.
D.  Despite seeming to promise more, the consumer civilization gives away exactly what the thing is worth.
E.  The consumer civilization attempts to spoil children and adults alike.

Ans: C
12. In the passage, the phrase “little parvenus” refers to
A.  naughty midgets
B.  old hags
C.  arrogant people
D.  young upstarts
E.  foolish kids
Ans: D
13. The author pined for two-cent cones instead of one four-cent pie because
A.  it made dietetic sense
B.  it suggested intemperance
C.  it was more fun
D.  it had a visual appeal
E.  he was a glutton
Ans: B
14. What does the author mean by “now a days the moralist risks seeming at odds with morality”?
A.  The moralist of yesterday have become immoral today
B.  The concept of morality has changed over the years
D.  The risks associated with immorality have gone up
E.  The purist’s view of morality is fast becoming popular
Ans: B
15. According to the author, the justification for refusal to let him eat two cones was plausibly
A.  didactic
B.  dietetic
C.  dialectic
D.  diatonic
E.  diastolic
Ans: A. 
Directions 11-15: In each of the following questions, out of the given alternatives, choose the one which       best expresses the meaning of the given word.
16. Ridicule
(A) recognize    (B) continuous    (C) gobble   (D) ludicrous
Ans: (C)
17. Gargantuan
(A) different   (B) removable  (C) colossal   (D) minuscule
Ans: (C)
 18. Savvy
(A) introduced   (B) underestimated  (C) knowledgeable (D) tremulous
Ans: (C)
19.  Abandonment
A. Care            B. Protection        C. Concern         D. Desertion
Ans: (D)
20. levied
imposed        believed        requested        correlated
Ans: A
Directions 21 to 25: Pick out from the words, given below each sentence, the  word which would  complete the sentence correctly and meaningfully.
21.The pot officer told me to wait, but I………………a year now and my parcel has not come yet.
    A) waited    B) am waiting    C) had waited    D) have waited.
    Ans: A
22. A blind man is no judge………….colours.
    A) of    B) for    C) in    D) of the
    Ans: C
23. He is……….. Minister of the Central Government
    A) a best and honest    B) the best and honest    C) best and the most honest    D) the best and most honest
    Ans: A
24. Over indulgence………….. character as well as physical stamina.
    A) strengthens    B) stimulate    C) debilitate    D) maintains
    Ans: B
25.Usually the ascent of a mountain face is much easier than the……………….
    A) fall    B) decent    C) descent    D) descend
    Ans: D
Directions for Questions 26-30: In each of the following questions there are sentences that form a paragraph. Identify the sentence’s or part’s of sentence’s that is/are correct in terms of grammar and usage (including spelling, punctuation and logical consistency). Then, choose the most appropriate option.
26.A. In 1849, a poor Bavarian immigrant named Levi Strauss
B. landed in San Francisco, California,
C. at the invitation of his brother-in-law David Stern
D. owner of dry goods business.
E. This dry goods business would later became known as Levi Strauss & Company.
A.  B only
B.  B and C
C.  A and B
D.  A only
E.  A, B and D
Ans. A
27.A. In response to the allegations and condemnation pouring in,
B. Nike implemented comprehensive changes in their labour policy.
C. Perhaps sensing the rising tide of global labour concerns,
D. from the public would become a prominent media issue,
E. Nike sought to be a industry leader in employee relations.
A.  D and E
B.  D only
C.  A and E
D.  A and D
E.  B, C and E
Ans: D
28.A. Charges and counter charges mean nothing
B. to the few million who have lost their home.
C. The nightmare is far from over, for the government
D. is still unable to reach hundreds who are marooned.
E. The death count have just begun.
A.  A only
B.  C only
C.  A and C
D.  A, C and D
E.  D only
Ans: D
29.A. I did not know what to make of you.
B. Because you’d lived in India, I associate you more with my parents than with me.
C. And yet you were unlike my cousins in Calcutta, who seem so innocent and obedient when I visited them
D. You were not curious about me in the least.
E. Although you did make effort to meet me.
A.  A only
B.  A and B
C.  A and E
D.  D only
E. A and D
Ans : E
30.A. I was asked to stop writing
B. She denied to go with me
C. My hairs stood on end
D. I am reading this novel for four days.
E. He as well as you are guilty.
A.  A only
B.  A and B
C.  A and E
D.  D only
E. A and D
Ans : A